Sei sulla pagina 1di 48

SECOND DIVISION

[G.R. No. 50464. January 29, 1990.]


SUNBEAM CONVENIENCE FOODS INC., CORAL
BEACH DEVELOPMENT CORP., and the REGISTER
OF
DEEDS
OF
BATAAN, petitioners, vs. HON.
COURT OF APPEALS and THE REPUBLIC OF THE
PHILIPPINES, respondents.
Filoteo T. Banzon for petitioners.
SYLLABUS
1. REMEDIAL LAW; PETITION FOR REVIEW ON CERTIORARI; GRANT THEREOF; NOT A
MATTER OF RIGHT. A petition for review on certiorari is not a matter of right but of
sound judicial discretion, and is granted only when there are special and important
reasons therefore. The following, while neither controlling nor fully measuring the
Court's discretion, enumerates the premises for granting a review: (a) When the
Court of Appeals has decided a question of substance, not theretofore determined
by the Supreme Court or has decided it in a way probably not in accord with law or
the applicable decisions of the Supreme Court; and (b) When the Court of Appeals
has so far departed from the accepted and usual course of judicial proceedings or
so far sanctioned such departure by a lower court as to call for supervision.
2. ID.; RULES OF PROCEDURE CONSTRUED; DESIGNED TO FACILITATE ATTAINMENT
OF JUSTICE. Generally, the rules of procedure must be observed so that the
efficient administration of justice is ensured. However, the rules of procedure should
be viewed as mere tools designed to facilitate the attainment of justice. They must
lead to the proper and just determination of litigation, without tying the hands of
the law or making it indifferent to realities.
3. ID.; SPECIAL CIVIL ACTION; CERTIORARI; GROUNDS FOR GRANT THEREOF.
Considered extraordinary, certiorari is made available only when there is no appeal,
nor any plain, speedy or adequate remedy in the ordinary course of the law. The
long line of decisions denying the petition for certiorari, either before appeal was
availed of or specially in instances where the appeal period has lapsed, far
outnumbers the instances when certiorari was given due course. The few significant
exceptions were: when public welfare and the advancement of public policy dictate;
or when the broader interests of justice so require, or when the writs issued are null,
or when the questioned order amounts to an oppressive exercise of judicial
authority.
4. LAND TITLES; FOREST LANDS; TO BE ALIENABLE REQUIRE, AN OFFICIAL
PROCLAMATION TO THAT EFFECT. Our adherence to the Regalian doctrine
subjects all agricultural, timber, and mineral lands to the dominion of the State.
Thus, before any land may be declassified from the forest group and converted into
alienable or disposable land for agricultural or other purposes, there must be a
positive act from the government. Even rules on the confirmation of imperfect titles

do not apply unless and until the land classified as forest land is released in an
official proclamation to that effect so that it may form part of the disposable
agricultural lands of the public domain. The mere fact that a title was issued by the
Director of Lands does not confer any validity on such title if the property covered
by the title or patent is part of the public forest.

DECISION

SARMIENTO, J p:
In this petition for review on certiorari, Sunbeam Convenience Foods Corporation
(hereafter simply SUNBEAM) and Coral Beach Development Corporation (hereafter
simply CORAL BEACH) bring to our attention the decision rendered by the Court of
Appeals in "Republic of the Philippines v. Hon. Pedro T. Santiago, et al.," disposing as
follows:
WHEREFORE, the writ prayed for is granted. The order of the
respondent judge dated October 7, 1977, dismissing Civil Case No.
4062 is set aside, and respondent judge is ordered to require private
respondents to file their answer to the complaint in said Civil Case
No. 4062 and thereafter to proceed with the trial of the case on the
merits and to render judgment thereon. 1
The following facts stated by the respondent Court in its decision and restated by
the petitioners in their petition are accurate:
a) On April 29, 1963, the Director of Lands caused the issuance of a
Sales Patent in favor of defendant Sunbeam Convenience Foods, Inc.,
over the parcels of land both situated in Mariveles, Bataan and more
particularly described and bounded as follows:
Lot 1-Sgs-2409 (area 3,113,695 sq. m.)
Lot 2-Sgs-2409 (area 1,401,855 sq. m.)
(b) On May 3, 1963, the aforesaid Sales Patent was registered with
the defendant Register of Deeds of Bataan who in turn issued Original
Certificate of Title No. Sp-24 in favor of
defendant SunbeamConvenience Foods, Inc., for the two parcels of
land above-described;
(c) Subsequently, Original Certificate of Title No. Sp-24 was cancelled
and in lieu thereof, Transfer Certificate of Title No. T-12421 was
issued over Lot 1, Sgs-2409, while Transfer Certificate of Title No.
12422 was issued over Lot 2, Sgs-2409, both in favor of defendant
Coral Beach Development Corporation;
(d) On May 11, 1976, the Solicitor General in the name of the
Republic of the Philippines instituted before the Court of First Instance
of Bataan, an action for reversion docketed as Civil Case No. 4062. 2
SUNBEAM and CORAL BEACH filed a Motion to Dismiss on the following grounds:
1. The Republic of the Philippines should have exhausted all
administrative remedies before filing the case in court;
2. The title issued to SUNBEAM and CORAL BEACH had become
indefeasible and imprescriptible;

3. The action for reversion was defective, having been initiated by


the Solicitor General and not by the Director of Lands. 3
The then Court of First Instance of Bataan dismissed the complaint in the Order of
October 7, 1977, 4 adopting mainly the theory that since the titles sought to be
cancelled emanated from the administrative act of the Bureau of Lands Director, the
latter, not the courts, had jurisdiction over the disposition of the land. LLphil
The Solicitor General received the copy of the Order on October 11, 1977 and filed a
Notice of Appeal dated October 25, 1977. 5 The Solicitor General then moved for an
extension of thirty days within which to file the Record on Appeal and to pay the
docket fee in order to perfect the appeal. This was to be followed by another motion
for extension filed by the Solicitor General, resulting in the Court of Appeals
granting the petitioner another extension of fifteen days from December 10, 1977.
Finally before this period of extension lapsed, instead of an appeal, a petition for
certiorari with the respondent Court of Appeals was filed.
According to the Solicitor General, the Court of First Instance committed grave
abuse of discretion in dismissing the complaint and in
a. Not finding that since the lower court acted in a Motion to Dismiss,
the correctness of its decision must be decided in the assumed truth
and accuracy of the allegations of the complaint. The complaint
alleges that the lands in question are forest lands; hence, inalienable.
b. Finding that Lots 1 and 2 are alienable and disposable lands of the
public domain under the jurisdiction of the Director of Lands despite
clear and positive evidence to the contrary.
c. Concluding that the complaint for reversion is defective as it was
not initiated by the Director of Lands.
d. Finding that the complaint for reversion states no cause of action
for alleged failure of petitioner to exhaust administrative remedies. 6
The Court of Appeals gave due course to the petition for certiorari, set aside the
Order of Dismissal rendered by the Court of First Instance in Civil Case No. 4062,
and ordered the presiding judge Hon. Pedro T. Santiago to receive the answers of
the private respondents SUNBEAM and CORAL BEACH in the action for reversion.
Hence Sunbeam and Coral Beach filed this petition for review. prLL
A review is not a matter of right but of sound judicial discretion, and is granted only
when there are special and important reasons therefore. The following, while neither
controlling nor fully measuring the Court's discretion, enumerates the premises for
granting a review:
(a) When the Court of Appeals has decided a question of substance,
not theretofore determined by the Supreme Court or has
decided it in a way probably not in accord with law or the
applicable decisions of the Supreme Court; and
(b) When the Court of Appeals has so far departed from the accepted
and usual course of judicial proceedings or so far sanctioned
such departure by a lower court as to call for supervision. 7

We agree with the Court of Appeals' granting of the petition filed by the Republic of
the Philippines charging the then Court of First Instance with grave abuse of
discretion.
The filing of the Motion to Dismiss the complaint for reversion by SUNBEAM and
CORAL BEACH on the ground of lack of cause of action, necessarily carried with it
the admission, for purposes of the motion, of the truth of all material facts pleaded
in the complaint instituted by the Republic.
An important factual issue raised in the complaint was the classification of the lands
as forest lands. This material allegation stated in the Republic's complaint 8 was
never denied specifically 9 by the defendants (petitioners herein) SUNBEAM and
CORAL BEACH.
If it is true that the lands are forest lands, then all these proceedings become moot
and academic. Land remains unclassified land until it is released therefrom and
rendered open to disposition. 10
Our adherence to the Regalian doctrine subjects all agricultural, timber, and mineral
lands to the dominion of the State. 11 Thus, before any land may be declassified
from the forest group and converted into alienable or disposable land for
agricultural or other purposes, there must be a positive act from the government.
Even rules on the confirmation of imperfect titles do not apply unless and until the
land classified as forest land is released in an official proclamation to that effect so
that it may form part of the disposable agricultural lands of the public domain. 12
The mere fact that a title was issued by the Director of Lands does not confer any
validity on such title if the property covered by the title or patent is part of the
public forest. 13
The only way to resolve this question of fact as to the classification of the land is by
remanding the case to the lower court for a full-dress trial on the issues involved.
Generally, the rules of procedure must be observed so that the efficient
administration of justice is ensured. However, the rules of procedure should be
viewed as mere tools designed to facilitate the attainment of justice. 14 They must
lead to the proper and just determination of litigation, without tying the hands of
the law or making it indifferent to realities. cdll
Certiorari is one such remedy. Considered extraordinary, it is made available only
when there is no appeal, nor any plain, speedy or adequate remedy in the ordinary
course of the law. 15 The long line of decisions denying the petition for certiorari,
either before appeal was availed of or specially in instances where the appeal
period has lapsed, far outnumbers the instances when certiorari was given due
course. The few significant exceptions were: when public welfare and the
advancement of public policy dictate; or when the broader interests of justice so
require, or when the writs issued are null, 16 or when the questioned order amounts
to an oppressive exercise of judicial authority. 17
We find nothing disagreeable with the action of the Court of Appeals to give due
course to the petition considering that the issue affected a matter of public concern
which is the disposition of the lands of our patrimony. No less than the Constitution
protects this policy. prLL

We therefore find no compelling reason to disturb the findings of the appellate


court, in the absence of a clear showing that the Court of Appeals has decided a
question of substance in a manner inconsistent with jurisprudence, or that the
respondent Court has departed from the accepted and usual course of judicial
proceedings. In sum, no reversible error has been committed by the respondent
court. 18
WHEREFORE, the petition is DENIED and the decision of the Court of Appeals is
affirmed. Costs against the petitioners.
SO ORDERED.
Melencio-Herrera, Paras, Padilla and Regalado, JJ., concur.
||| (Sunbeam Convenience Foods Inc. v. Court of Appeals, G.R. No. 50464, [January
29, 1990], 260 PHIL 470-477)

FIRST DIVISION
[G.R. No. 60413. October 31, 1990.]
REPUBLIC
OF
THE
PHILIPPINES, petitioner, vs. HON.
SOFRONIO
G. SAYO, Judge, Br. I, CFI, Nueva Vizcaya, HEIRS
OF CASIANO SANDOVAL, HEIRS OF LIBERATO
BAYAUA, JOSE C. REYES, and PHILIPPINE CACAO
AND FARM PRODUCTS INC., respondents.
Celso D. Gangan respondent Heirs of Liberato Bayaua.
Acosta & Associates for Phil. Cacao and Farm Products, Inc.
Jose Reyes & Associates for Heirs of Casiano Sandoval, et al.

DECISION

NARVASA, J p:
Sought to be annulled and set aside in this special civil action of certiorari is the
decision of respondent Judge Sofronio G. Sayo rendered on March 5, 1981 in Land
Registration Case No. N-109, LRC Record No. 20850, confirming, by virtue of a
compromise agreement, the title of the private respondents over a tract of
land. LLjur
The spouses, Casiano Sandoval and Luz Marquez, filed an original application for
registration of a tract of land identified as Lot No. 7454 of the Cadastral Survey of
Santiago, BL Cad. 211 (July 17, 1961) and having an area of 33,950 hectares. The

land was formerly part of the Municipality of Santiago, Province of Isabela, but had
been transferred to Nueva Vizcaya in virtue of Republic Act No. 236.
Oppositions were filed by the Government, through the Director of Lands and the
Director of Forestry, and some others, including the Heirs of Liberato Bayaua. 1 In
due course, an order of general default was thereafter entered on December 11,
1961 against the whole world except the oppositors.
The case dragged on for about twenty (20) years until March 3, 1981 when a
compromise agreement was entered into by and among all the parties, assisted by
their respective counsel, namely: the Heirs of Casiano Sandoval (who had since
died), the Bureau of Lands, the Bureau of Forest Development, the Heirs of Liberato
Bayaua, and the Philippine Cacao and Farm Products, Inc. Under the compromise
agreement, the Heirs of Casiano Sandoval (as applicants) renounced their claims
and ceded
1) in favor of the Bureau of Lands, an area of 4,109 hectares;
2) in favor of the Bureau of Forest Development, 12,341 hectares;
3) in favor of the Heirs of Liberato Bayaua, 4,000 hectares; and
4) in favor of Philippine Cacao & Farm Products, Inc., 8,000 hectares.
The remaining area of 5,500 hectares was, under the compromise agreement,
adjudicated to and acknowledged as owned by the Heirs of Casiano Sandoval, but
out of this area, 1,500 hectares were assigned by the Casiano Heirs to their counsel,
Jose C. Reyes, in payment of his attorney's fees. In consideration of the areas
respectively allocated to them, all the parties also mutually waived and renounced
all their prior claims to and over Lot No. 7454 of the Santiago Cadastre.
In a decision rendered on March 5, 1981, the respondent Judge approved the
compromise agreement and confirmed the title and ownership of the parties in
accordance with its terms.
The Solicitor General, in behalf of the Republic of the Philippines, has taken the
present recourse in a bid to have that decision of March 5, 1981 annulled as being
patently void and rendered in excess of jurisdiction or with grave abuse of
discretion. The Solicitor General contends that
1) no evidence whatever was adduced by the parties in support of their petitions for
registration;
2) neither the Director of Lands nor the Director of Forest Development had legal
authority to enter into the compromise agreement;
3) as counsel of the Republic, he should have been but was not given notice of the
compromise agreement or otherwise accorded an opportunity to take part therein;
4) that he was not even served with notice of the decision approving the
compromise; it was the Sangguniang Panlalawigan of Quirino Province that drew his
attention to the "patently erroneous decision" and requested him to take immediate
remedial measures to bring about its annulment.
The respondents maintain, on the other hand, that the Solicitor General's
arguments are premised on the proposition that Lot 7454 is public land, but it is not.

According to them, as pointed out in the application for registration, the private
character of the land is demonstrated by the following circumstances, to wit:
1) the possessory information title of the applicants and their predecessors-ininterest;
2) the fact that Lot 7454 was never claimed to be public land by the Director of
Lands in the proper cadastral proceedings;
3) the pre-war certification of the National Library dated August 16, 1932 to the
effect that the Estadistica de Propiedades of Isabela issued in 1896 and appearing
in the Bureau of Archives, the property in question was registered under the
'Spanish system of land registration as private property owned by Don Liberato
Bayaua, applicants' predecessors-in-interest;
4) the proceeding for registration, brought under Act 496 (the Torrens Act)
presupposes that there is already a title to be confirmed by the court, distinguishing
it from proceedings under the Public Land Act where the presumption is always that
the land involved belongs to the State.
Under the Regalian Doctrine, 2 all lands not otherwise appearing to be clearly within
private ownership are presumed to belong to the State. Hence it is that all
applicants in land registration proceedings have the burden of overcoming the
presumption that the land thus sought to be registered forms part of the public
domain. 3 Unless the applicant succeeds in showing by clear and convincing
evidence that the property involved was acquired by him or his ancestors either by
composition title from the Spanish Government or by possessory information title,
or any other means for the proper acquisition of public lands, the property must be
held to be part of the public domain. 4 The applicant must present competent and
persuasive proof to substantiate his claim; he may not rely on general statements,
or mere conclusions of law other than factual evidence of possession and title. 5
In the proceeding at bar, it appears that the principal document relied upon and
presented by the applicants for registration, to prove the private character of the
large tract of land subject of their application, was a photocopy of a certification of
the National Library dated August 16, 1932 (already above mentioned) to the effect
that according to the Government's Estadistica de Propiedades of Isabela issued in
1896, the property in question was registered under the Spanish system of land
registration as private property of Don Liberato Bayaua. But, as this Court has
already had occasion to rule, that Spanish document, the Estadistica de
Propiedades, cannot be considered a title to property, it not being one of the grants
made during the Spanish regime, and obviously not constituting primary evidence
of ownership. 6 It is an inefficacious document on which to base any finding of the
private character of the land in question.
And, of course, to argue that the initiation of an application for registration of land
under the Torrens Act is proof that the land is of private ownership, not pertaining to
the public domain, is to beg the question. It is precisely the character of the land as
private which the applicant has the obligation of establishing. For there can be no
doubt of the intendment of the Land Registration Act, Act 496, that every applicant
show a proper title for registration; indeed, even in the absence of any adverse
claim, the applicant is not assured of a favorable decree by the Land Registration
Court, if he fails to establish a proper title for official recognition.

It thus appears that the decision of the Registration Court a quo is based solely on
the compromise agreement of the parties. But that compromise agreement included
private persons who had not adduced any competent evidence of their ownership
over the land subject of the registration proceeding. Portions of the land in
controversy were assigned to persons or entities who had presented nothing
whatever to prove their ownership of any part of the land. What was done was to
consider the compromise agreement as proof of title of the parties taking part
therein, a totally unacceptable proposition. The result has been the adjudication of
lands of no little extension to persons who had not submitted any substantiation at
all of their pretensions to ownership, founded on nothing but the agreement among
themselves that they had rights and interests over the land. cdll
The assent of the Directors of Lands and Forest Development to the compromise
agreement did not and could not supply the absence of evidence of title required of
the private respondents.
As to the informacion posesoria invoked by the private respondents, it should be
pointed out that under the Spanish Mortgage Law, it was considered a mode of
acquiring title to public lands, subject to two (2) conditions: first, the inscription
thereof in the Registry of Property, and second, actual, public, adverse, and
uninterrupted possession of the land for twenty (20) years (later reduced to ten [10]
years); but where, as here, proof of fulfillment of these conditions is absent,
the informacion posesoria cannot be considered as anything more than prima
facie evidence of possession. 7
Finally, it was error to disregard the Solicitor General in the execution of the
compromise agreement and its submission to the Court for approval. It is, after all,
the Solicitor General, who is the principal counsel of the Government; this is the
reason for our holding that "Court orders and decisions sent to the fiscal, acting as
agent of the Solicitor General in land registration cases, are not binding until they
are actually received by the Solicitor General." 8
It thus appears that the compromise agreement and the judgment approving it
must be, as they are hereby, declared null and void, and set aside. Considerations
of fairness however indicate the remand of the case to the Registration Court so
that the private parties may be afforded an opportunity to establish by competent
evidence their respective claims to the property. Cdpr

WHEREFORE, the decision of the respondent Judge complained of is ANNULLED and


SET ASIDE. Land Registration Case No. N-109 subject of the petition is REMANDED
to the court of origin which shall conduct further appropriate proceedings therein,
receiving the evidence of the parties and thereafter rendering judgment as such
evidence and the law may warrant. No pronouncement as to costs.
SO ORDERED.
Cruz, Gancayco, Grio-Aquino and Medialdea, JJ., concur.
||| (Republic v. Sayo, G.R. No. 60413, [October 31, 1990], 269 PHIL 74-80)

THIRD DIVISION
[G.R. No. 129401. February 2, 2001.]
FELIPE
SEVILLE
in
his
capacity
as
judicial
administrator of the estate of JOAQUIN ORTEGA
and/or FELIPE SEVILLE, EMILIA ESTRADA, MARIA S.
TELLDER, MA. ISABEL SEVILLE, MA. TERESITA
LICARDO, FRANCISCO SEVILLE, RAMON O. SEVILLE,
JOSE MARIE SEVILLE, GEMMA ALVAREZ-ASAYAS,
ANNABELLE ALVAREZ-GONZALES, SYLVIA ALVAREZ-

LIOK, ADOLFO O. ALVAREZ JR., DIANA ALVAREZDABON, MARIA SALVADOR O. POLANCOS and JOAQUIN
ORTEGA II as successors-in-interest of JOAQUIN
ORTEGA and his estate, petitioners, vs. NATIONAL
DEVELOPMENT
COMPANY,
LEYTE
SAB-A
BASIN
DEVELOPMENT AUTHORITY, PHILIPPINE ASSOCIATED
SMELTING AND REFINING CORPORATION, LEPANTOCONSOLIDATED MINING CO., PHILIPPINE PHOSPHATE
FERTILIZER
CORPORATION,
CALIXTRA
YAP
and
REGISTER OF DEEDS OF LEYTE, respondents.
Arthur D. Lim Law Office for petitioners.
Government Corporate Counsel for respondent NDC & LSDA.
Castillo Laman Tan Pantaleon & San Jose for respondent PASAR & LEPANTO.
Quiros Dumas Capistrano & Teleron, Law Offices for PHILPOS.
SYNOPSIS
Leyte Sab-A Basin Development Authority (LSBDA) was created to integrate
government and private sector efforts for a planned development and balanced
growth of the Sab-A Basin in the Province of Leyte, empowered to acquire real
property in the successful prosecution of its business. Respondent Calixtra Yap sold
to LSBDA Lot No. 057 SWO 08-000047 consisting of 464,920- sq. m. Original
Certificate of Title was issued for the said property in the name of respondent
LSBDA in 1983. In 1989, LSBDA assigned all its rights over the subject property to
its co-respondent National Development Company (NDC), as a result of which a new
Transfer Certificate of Title was issued in the name of NDC. Meanwhile also in 1989,
the Estate of Joaquin Ortega, represented by administrator Felipe Seville filed with
the Regional Trial Court an action for the recovery of real property, rentals and
damages against all the respondents herein. The Estate claimed ownership by
acquisitive prescription over the real property acquired by LSBDA and later by NDC,
containing a land area of 735,333-sq. m., which included that portion sold by
Calixtra Yap. The trial court rendered its decision in favor of the petitioners herein
and against the respondents. Respondents herein appealed their case to the Court
of Appeals (CA), which in turn reversed the decision of the trial court. The Court of
Appeals citing the Regalian doctrine ruled that the land in question, which did not
appear to be privately owned, should be presumed as part of the public domain.
Hence, petitioners filed a Petition for Review on Certiorari before the Supreme Court
after the CA denied their Motion for Reconsideration. The Court was called upon to
determine the validity of LSBDA's title. In resolving the issue, the Court has to rule
on whether the land in question was private or public before the issuance of title
thereof.
The Supreme Court ruled in favor of the respondents herein and affirmed the ruling
of the Court of Appeals. According to the Court there was no showing that the land

had been classified as alienable before the title was issued to LSBDA; hence,
petitioners could not be the owners thereof through acquisitive prescription.
Moreover, LSBDA's title was derived from a miscellaneous sales patent, not from
Yap. The Court ruled that the petitioner's challenge to LSDBA's title cannot be
granted since a certificate that had already became indefeasible and
incontrovertible cannot be challenged by a collateral attack.

SYLLABUS
1. POLITICAL LAW; LANDS OF THE PUBLIC DOMAIN; REGALIAN DOCTRINE; DEFINED
AND CONSTRUED; ACQUISITION OF IMPERFECT TITLE TO PUBLIC LANDS;
REQUISITES THEREOF; APPLICABLE ONLY TO ALIENABLE AND DISPOSABLE LANDS
OF THE PUBLIC DOMAIN. Under the Regalian doctrine, all the lands of the public
domain belong to the State, which is the source of any asserted right to ownership
of land. All lands not otherwise appearing to be clearly within private ownership are
presumed to belong to the State. In Menguito v. Republic, the court held that
"[u]nless public land is shown to have been reclassified or alienated to a private
person by the State, it remains part of the inalienable public domain. Indeed,
'occupation thereof in the concept of owner, no matter how long, cannot ripen into
ownership and be registered as a title.' To overcome such presumption,
incontrovertible evidence must be shown by the applicant. Absent such evidence,
the land sought to be registered remains inalienable." A person in open, continuous,
exclusive and notorious possession of a public land for more than thirty years
acquires an imperfect title thereto. That title may be the subject of judicial
confirmation, pursuant to Section 48 of the Public Land Act, which provides:
"SECTION 48. The following described citizens of the Philippines, occupying lands of
public domain or claiming to own any such lands or an interest thereon, but whose
titles have not been perfected or completed, may apply to the Court of First
Instance of the province where the land is located for confirmation of their claims,
and the issuance of a certificate of title therefore, under the Land Registration Act,
to wit: . . . (b) those who by themselves or through their predecessor-in-interest
have been in open, continuous, exclusive and notorious possession and occupation
of agricultural lands of the public domain, under a bona fide claim of acquisition or
ownership, for at least thirty years immediately preceding the filing of the
application for confirmation of title except when prevented by war orforce majeure.
They shall be conclusively presumed to have performed all the conditions essential
to a Government grant and shall be entitled to a certificate of title under the
provisions of this Chapter." Under Section 4 of Presidential Decree (PD) No. 1073,
paragraph "b" of the aforecited provision applies only to alienable and disposable
lands of the public domain. The provision reads: "SEC. 4. The provisions of Section
48 (b) and Section 48 (c), Chapter VIII, of the Public Land Act, are hereby amended
in the sense that these provisions shall apply only to alienable and disposable lands
of the public domain which have been in open, continuous, exclusive and notorious
possession and occupation by the applicant himself or thru his predecessor-ininterest, under a bona fide claim of acquisition of ownership, since June 12, 1945." It
should be stressed that petitioners had no certificate of title over the disputed
property. Although they claim that their title was based on acquisitive prescription,
they fail to present incontrovertible proof that the land had previously been

classified as alienable. They simply brush aside the conclusion of the CA on this
crucial point by saying that it was "without factual basis." Instead, they maintain
that the private character of the land was evidenced by various tax declarations,
Deeds of Sale, and Decisions of the trial court and even the Supreme Court.
2. CIVIL LAW; LAND REGISTRATION; TAX DECLARATION; NOT CONCLUSIVE PROOF OF
LAND OWNERSHIP NOR THAT THE LAND IS PRIVATE; APPLICATION IN CASE AT BAR.
Tax declarations are not conclusive proofs of ownership, let alone of the private
character of the land. At best, they are merely "indicia of a claim of ownership."
In Spouses Palomo vs. CA, the court also rejected tax declarations as proof of
private ownership, absent any showing that the forest land in question had been
reclassified as alienable. Moreover, the Deeds of Sale of portions of the disputed
property, which Joaquin Ortega and several vendors executed, do not prove that the
land was private in character. The question remains: What was the character of the
land when Ortega purchased it? Indeed, a vendee acquires only those rights
belonging to the vendor. But petitioners failed to show that, at the time, the vendors
were already its owners, or that the land was already classified as alienable.
3. ID.; ID.; BURDEN OF PROOF THAT THE LAND HAD BEEN CLASSIFIED AS ALIENABLE
IS ON THE CLAIMANT; ABSENCE THEREOF IN CASE AT BAR. Clearly, the burden of
proof that the land has been classified as alienable is on the claimant. In the present
case, petitioners failed to discharge this burden. Hence, their possession of the
disputed property, however long, cannot ripen into ownership.
4. ID.; ID.; WHEN CERTIFICATE OF TITLE OF LAND OF PUBLIC DOMAIN MAY BE
ISSUED; CASE AT BAR. Equally unmeritorious is the argument of petitioners that
the title of LSBDA is void. As earlier stated, they claim that such title was derived
from Calixtra Yap, who was allegedly not the owner of the property. Petitioners
assume that LSBDA, having acquired the rights of Yap, resorted to a confirmation of
her imperfect title under Section 48 of the Public Land Act. This argument is devoid
of factual or legal basis. Petitioners fail to consider that the title of LSBDA was
based, not on the conveyance made by Yap, but on Miscellaneous Sales Patent No.
9353 issued by the director of the Bureau of Lands. In fact, after LSBDA had filed an
application for patent, the Bureau of Lands conducted an investigation and found
that the land was part of the public domain. After compliance with the notice and
publication requirements, LSBDA acquired the property in a public auction
conducted by the Bureau of Lands. In any case, the actions of LSBDA after Yap's
conveyance demonstrated its position that the disputed land was part of the public
domain. That this was so can be inferred from LSBDA's subsequent application for a
Miscellaneous Sales Patent and, in a public auction, its purchase of the property
from the Bureau of Lands. Indeed, Yap merely conveyed a claim, not a title which
she did not have.
5. ID.; ID.; CERTIFICATE OF TITLE; NATURE THEREOF; MAY BE QUESTIONED ONLY IN
A DIRECT PROCEEDING PERMITTED BY LAW. It has been held that a certificate of
title, once registered, should not thereafter be impugned, altered, changed,
modified, enlarged or diminished, except in a direct proceeding permitted by law.
Otherwise, the reliance on registered titles would be lost. Moreover, the title
became indefeasible and incontrovertible after the lapse of one year from the time
of its registration and issuance. Section 32 of PD 1529 provides that "[u]pon the
expiration of said period of one year, the decree of registration and the certificate of
title shall become incontrovertible. Any person aggrieved by such decree of

registration in any case may pursue his remedy by action for damages against the
applicant or other persons responsible for the fraud." Although LSBDA's title was
registered in 1983, petitioners filed the amended Complaint only in 1990.

6. ID.;.ID.; ID.; RECONVEYANCE THEREOF; NOT ALLOWED IN CASE, AT BAR.


Petitioners also claim that the disputed property should be reconveyed to them.
This cannot be allowed. Considering that the land was public before the
Miscellaneous Sales Patent was issued to LSBDA, petitioners have no standing to
ask for the reconveyance of the property to them. The proper remedy is an action
for reversion, which may be instituted only by the Office of the Solicitor General,
pursuant to Section 101 of the Public Land Act, which reads as follows: "SEC. 101.
All actions for the reversion to the Government of lands of the public domain or
improvements thereon shall be instituted by the Solicitor General or the officer
acting in his stead, in the proper courts, in the name of the [Republic] of the
Philippines." Verily, the prayer for reconveyance and, for that matter, the entire
case of petitioners rest on the theory that they have acquired the property by
acquisitive prescription; and that Yap, without any right or authority, sold the same
to LSBDA. TCcDaE

DECISION
PANGANIBAN, J p:
Unless a public land is shown to have been reclassified as alienable or actually
alienated by the State to a private person, that piece of land remains part of the
public domain. Hence, occupation thereof, however long, cannot ripen into
ownership.
The Case
Before us is a Petition for Review on Certiorari assailing the November 29, 1996
Decision of the Court of Appeals 1 (CA), as well as the May 19, 1997 CA
Resolution 2 denying the Motion for Reconsideration. The dispositive part of the CA
Decision reads as follows:
"WHEREFORE, the appealed decision is REVERSED and SET ASIDE.
Another judgment is hereby rendered dismissing the complaint. 'The
counterclaim of appellants are denied. Costs against plaintiffsappellees." 3
The Facts
The appellate court narrated the undisputed facts in this manner:
"1. By virtue of Presidential Decree No. 625, Leyte Sab-A Basin
Development Authority (LSBDA) was created to integrate government
and private sector efforts for a planned development and balanced
growth of the Sab-a Basin in the [P]rovince of Leyte, empowered to
acquire real property in the successful prosecution of its
business. Letter of Instruction No. 962 authorized LSBDA to acquire
privately-owned lands circumscribed in the Leyte Industrial
Development Estate (LIDE) by way of negotiated sales with the
landowners.

"2. On June 14; 1980, [Respondent] Calixtra Yap sold to LSBDA Lot
No. 057 SWO 08-000047 consisting of 464,920 square meters,
located at Barangay Sto. Rosario, Isabel, Leyte, covered under Tax
Declarations Nos. 3181, 3579, 3425, 1292 and 4251 under the name
of said vendor.
"3. On June 1, 1982, appellant LSBDA filed a Miscellaneous Sales
Application with the Bureau of Lands covering said lot together with
other lots acquired by LSBDA with an aggregate area of '442,7508
square meters.'
"4. After due notice and investigation conducted by the Bureau of
Lands, Miscellaneous Sales Patent No. 9353 was issued in the name
of [Respondent] LSBDA on the basis of which Original Certificate of
Title No. P-28131 was transcribed in the Registration Book for the
[P]rovince of Leyte on August 12, 1983 in the name of [Respondent]
LSBDA. On December 14, 1989, LSBDA assigned all its rights over the
subject property to its [Co-respondent] National Development
Company (NDC) as a result of which a new Transfer Certificate of Title
was issued on March 2, 1990 by the Registry of Deeds for the
Province of Northern Leyte in the name of NDC. The subject property
was leased to [Respondents] Philippine Associated Smelting &
Refining Corporation (PASAR), Philippine Phosphate Fertilizer
Corporation (PHILPHOS) and Lepanto Consolidated Mining Co., Inc.
(LEPANTO).
"5. On November 29, 1988, the Estate of Joaquin Ortega represented
by judicial administrator Felipe Seville filed with the Regional Trial
Court (Branch 12) of Ormoc City, a complaint for recovery of real
property, rentals and damages against the above-named
[respondents] which complaint was later on amended on May 11,
1990. [Respondents] filed their respective Answers. After trial, the
trial court rendered judgment the dispositive portion of which reads
as follows:
'WHEREFORE, [a] decision is hereby rendered for [petitioners]
and against [respondents].
'1. The Deed of Sale executed by Calixtra Yap on June 14, 1980
in favor of LSBDA, (Exhibit PP and 25) conveying the subject
property to said LSBDA is declared NULL and VOID ab initio;
'2. The intestate estate of JOAQUIN ORTEGA is declared the
owner in fee simple of the 735,333 square meters real
property subject of the present action and defendant NDC is
ordered to segregate the same area from OCT P-28131 and
CONVEY the same to the Estate of Joaquin Ortega;
'3. Upon the segregation of the 735,333 square meters from
OCT No. P-28131 the Register of Deeds of the Province of Leyte
is ordered to issue a new title to the said portion in the name
of the Intestate Estate of Joaquin Ortega;

'4. [Respondents] LSBDA, NDC, PASAR, are ordered to pay


jointly and severally to [petitioners] the sum of FOUR MILLION
SEVEN HUNDRED EIGHTY FOUR THOUSAND EIGHT HUNDRED
FORTY SIX PESOS (P4,784,846.00) as rentals due from 1979 to
the present, plus accrued interest pursuant to par. 2 of the
Lease Contract between NDC and PASAR. (Exhibit 54)
'5. [Respondents] LSBDA, NDC, and PHILPHOS are also ordered
to pay jointly and severally [petitioners] the sum of TWO
MILLION EIGHTY SIX THOUSAND THREE HUNDRED NINETY
EIGHT PESOS AND SIXTY CENTAVOS (P2,086,398.60) as
accrued rentals of PHILPHOS from 1979 to present, plus the
accrued interest for non-payment pursuant to paragraph 2 of
the same Lease Contract cited above;
'6. [Respondents] are ordered to pay jointly and severally
[petitioners] P200,000.00 as indemnity for the value of the
ancestral home;
'7. [Respondents] are also ordered to pay jointly and severally
[petitioners] the sum of P250,000.00 as reimbursement for
attorney's fees and the further sum of P50,000.00 as expenses
for litigation;
'8. Finally, [petitioners] and [respondents] are ordered to sit
down together and discuss the possibility of a compromise
agreement on how the improvements introduced on the
landholding subject of the present suit should be disposed of
and for the parties to submit to this Court a joint manifestation
relative thereto. In the absence of any such compromise
agreement, such improvements shall be disposed of pursuant
to Article 449 of the New Civil Code.
'Costs against [respondents].
'SO ORDERED.'" 4
Ruling of the Court of Appeals
Citing the Regalian doctrine that lands not appearing to be privately owned are
presumed to be part of the public domain, the CA held that, first, there was no
competent evidence to prove that the property in question was private in
character. Second, possession thereof, no matter how long, would not ripen into
ownership, absent any showing that the land had been classified as
alienable. Third, the property had been untitled before the issuance of the
Miscellaneous Sales Patent in favor of LSBDA. Fourth, petitioners were guilty of
laches, because they had failed to apply for the judicial confirmation of their title, if
they had any. Fifth, there was no evidence of bad faith on the part of LSBDA in
dealing with Yap regarding the property.
Hence, this Petition. 5
The Issues

In their Memorandum, petitioners submit the following issues for the consideration
of the Court: 6
"A. Whether or not the sale by Calixtra Yap of the Estate of the Late
Joaquin Ortega in favor of LSBDA was null and void.
"B. Whether or not the issuance of a Miscellaneous Sales Patent and
an Original Certificate of Title in favor of LSBDA was valid.
"C. Whether or not petitioners are guilty of laches.
"D. Whether or not petitioners are entitled to the remedy of
reconveyance and the damages awarded by the trial court."
In the main, the Court is called upon to determine the validity of LSBDA's title. In
resolving this issue, it will also ascertain whether, before the issuance of the title,
the land was private or public.
The Court's Ruling
The Petition has no merit.
Main Issue: Validity of LSBDA's Title
Petitioners argue that LSBDA's title to 73 hectares of the 402-hectare Leyte
Industrial Development Estate was void, having allegedly been obtained from
Calixtra Yap who had no right to it. They maintain that they acquired title to the
disputed property by acquisitive prescription, because they and their predecessors
in interest had been in possession of it for more than thirty years. 7 Although it was
the subject of settlement proceedings, petitioners further claim that Yap sold the
same to LSBDA without the permission of the trial court.
Disputing these contentions, respondents and the appellate court maintain that
petitioners have not shown that the land had previously been classified as alienable
and disposable. Absent such classification, they argue that possession of it, no
matter how long, could not ripen into ownership.
We agree with respondents and the appellate court. First, there was no showing that
the land had been classified as alienable before the title was issued to LSBDA;
hence, petitioners could not have become owners thereof through acquisitive
prescription. Second, petitioners' challenge to LSBDA's title cannot be granted,
because it is based on a wrong premise and amounts to a collateral attack, which is
not allowed by law.
Public Character of the Land
Under the Regalian doctrine, all lands of the public domain belong to the State,
which is the source of any asserted right to ownership of land. All lands not
otherwise appearing to be clearly within private ownership are presumed to belong
to the State. 8 In Menguito v. Republic, 9 the Court held that "[u]nless public land is
shown to have been reclassified or alienated to a private person by the State, it
remains part of the inalienable public domain. Indeed, 'occupation thereof in the
concept of owner, no matter how long, cannot ripen into ownership and be
registered as a title.' To overcome such presumption, incontrovertible evidence must
be shown by the applicant. Absent such evidence, the land sought to be registered
remains inalienable." CDEaAI

A person in open, continuous, exclusive and notorious possession of a public land


for more than thirty years acquires an imperfect title thereto. That title may be the
subject of judicial confirmation, pursuant to Section 48 of the Public Land Act, which
provides:
"SECTION 48. The following described citizens of the Philippines,
occupying lands of public domain or claiming to own any such lands
or an interest thereon, but whose titles have not been perfected or
completed, may apply to the Court of First Instance of the province
where the land is located for confirmation of their claims, and the
issuance of a certificate of title therefor, under the Land Registration
Act, to wit:
xxx xxx xxx
(b) those who by themselves or through their predecessor in-interest
have been in open, continuous, exclusive and notorious possession
and occupation of agricultural lands of the public domain, under a
bona fide claim of acquisition or ownership, for at least thirty years
immediately preceding the filing of the application for confirmation of
title except when prevented by war or force majeure. They shall be
conclusively presumed to have performed all the conditions essential
to a Government grant and shall be entitled to a certificate of title
under the provisions of this Chapter."
Under Section 4 of Presidential Decree (PD) No. 1073, 10 paragraph "b" of the
aforecited provision applies only to alienable and disposable lands of the public
domain. The provision reads:
"SECTION 4. The provisions of Section 48 (b) and Section 48 (c),
Chapter VIII, of the Public Land Act, are hereby amended in the sense
that these provisions shall apply only to alienable and disposable
lands of the public domain which have been in open, continuous,
exclusive and notorious possession and occupation by the applicant
himself or thru his predecessor-in-interest, under a bona fide claim of
acquisition of ownership, since June 12, 1945."
It should be stressed that petitioners had no certificate of title over the disputed
property. Although they claim that their title was based on acquisitive prescription,
they fail to present incontrovertible proof that the land had previously been
classified as alienable. They simply brush aside the conclusion of the CA on this
crucial point by saying that it was "without factual basis." 11 Instead, they maintain
that the private character of the land was evidenced by various tax declarations,
Deeds of Sale, and Decisions of the trial court and even the Supreme Court. 12
Petitioners' arguments are not convincing. Tax declarations are not conclusive
proofs of ownership, let alone of the private character of the land. At best, they are
merely "indicia of a claim of ownership." 13 InSpouses Palomo v. CA, 14 the Court
also rejected tax declarations as proof of private ownership, absent any showing
that the forest land in question had been reclassified as alienable.

Moreover, the Deeds of Sale of portions of the disputed property, which Joaquin
Ortega and several vendors executed, do not prove that the land was private in
character. The question remains: What was the character of the land when Ortega
purchased it? Indeed, a vendee acquires only those rights belonging to the vendor.
But petitioners failed to show that, at the time, the vendors were already its owners,
or that the land was already classified as alienable.
Also misplaced is petitioners' reliance on Ortega v. CA, 15 in which the Supreme
Court allegedly recognized the private character of the disputed property. In that
case, the sole issue was "whether the respondent judge . . . acted in excess of
jurisdiction when he converted Civil Case No. 1184-O, an action for quieting of title,
declaration of nullity of sale, and annulment of tax declaration of a parcel of land,
into an action for the declaration of who is the legal wife, who are the legitimate
children, if any, and who are the compulsory heirs of the deceased Joaquin
Ortega." 16 The Court did not at all make any ruling that the property had been
classified as alienable.
In any event, Ortegaarose from a suit for quieting of title, an action quasi in
rem that was binding only between the parties. 17 The present respondents as well
as the Bureau of Lands, which subsequently declared that the land was public, are
not bound by that ruling, because they were not impleaded therein.
While petitioners refer to the trial court proceedings supposedly recognizing the
private character of the disputed property, they make no claim that these cases
directly involve the classification of the land, or that the Bureau of Lands is a party
thereto.
Clearly, the burden of proof that the land has been classified as alienable is on the
claimant. 18 In the present case, petitioners failed to discharge this burden. Hence,
their possession of the disputed property, however long, cannot ripen into
ownership.
LSBDA's Title
Equally unmeritorious is the argument of petitioners that the title of LSBDA is void.
As earlier stated, they claim that such title was derived from Calixtra Yap, who was
allegedly not the owner of the property. Petitioners assume that LSBDA, having
acquired the rights of Yap, resorted to a confirmation of her imperfect title under
Section 48 of the Public Land Act. This argument is devoid of factual or legal basis.
Petitioners fail to consider that the title of LSBDA was based, not on the conveyance
made by Yap, but on Miscellaneous Sales Patent No. 9353 issued by the director of
the Bureau of Lands. In fact, after LSBDA had filed an application for patent, the
Bureau of Lands conducted an investigation and found that the land was part of the
public domain. After compliance with the notice and publication requirements,
LSBDA acquired the property in a public auction conducted by the Bureau of
Lands. 19
Petitioners insist, however, that LSBDA was estopped from claiming that the land
was public, because the Deed of Sale executed by Yap in its favor stipulated that
"the seller is the absolute owner in fee simple of the . . . described property." 20 It is
scarcely necessary to address this point. To begin with, the power to classify a land
as alienable belongs to the State, not to private entities. Hence, the
pronouncements of Yap or LSBDA cannot effect the reclassification of the property.

Moreover, the assailed misrepresentation was made by Yap as seller. Hence,


objections thereto should be raised not by petitioners but by LSBDA. the contracting
party obviously aggrieved.
In any case, the actions of LSBDA after Yap's conveyance demonstrated its position
that the disputed land was part of the public domain. That this was so can be
inferred from LSBDA's subsequent application for a Miscellaneous Sales Patent and,
in a public auction, its purchase of the property from the Bureau of Lands. Indeed,
Yap merely conveyed a claim, not a title which she did not have.
Collateral Attack
There is another reason for denying the present Petition. Petitioners insist that they
"are not seeking the re-opening of a decree under the Torrens system." Supposedly,
they are only "praying for the segregation of 735,333 square meters of land, or 73
hectares more or less from the OCT No. P-28131 issued to LSBDA." 21 This
disputation is mere quibbling over words, plain and simple.
Semantics aside, petitioners are effectively seeking the modification of LSBDA's
OCT, which allegedly encompassed even a parcel of land allegedly belonging to
them. Hence, the present suit, purportedly filed for the "recovery of real property
and damages," is tantamount to a collateral attack not sanctioned by law. Section
48 of PD 1529, the Property Registration Decree, expressly provides:
"SECTION 48. Certificate not subject to collateral attack. A
certificate of title shall not be subject to collateral attack. It cannot be
altered, modified, or cancelled except in a direct proceeding in
accordance with law."
It has been held that a certificate of title, once registered, should not thereafter be
impugned, altered, changed, modified, enlarged or diminished, except in a direct
proceeding permitted by law. Otherwise, the reliance on registered titles would be
lost. 22
Moreover, the title became indefeasible and incontrovertible after the lapse of one
year from the time of its registration and issuance. 23 Section 32 of PD
1529 provides that "[u]pon the expiration of said period of one year, the decree of
registration and the certificate of title shall become incontrovertible. Any person
aggrieved by such decree of registration in any case may pursue his remedy by
action for damages against the applicant or other persons responsible for the
fraud." Although LSBDA's title was registered in 1983, petitioners filed the amended
Complaint only in 1990.
Reconveyance
Petitioners also claim that the disputed property should be reconveyed to them.
This cannot be allowed. Considering that the land was public before the
Miscellaneous Sales Patent was issued to LSBDA, petitioners have no standing to
ask for the reconveyance of the property to them. The proper remedy is an action
for reversion, which may be instituted only by the Office of the Solicitor General,
pursuant to Section 101 of the Public Land Act, which reads as follows:
"SECTION 101. All actions for the reversion to the Government of
lands of the public domain or improvements thereon shall be

instituted by the Solicitor General or the officer acting in his stead, in


the proper courts, in the name of the [Republic] of the Philippines."
Verily, the prayer for reconveyance and, for that matter, the entire case of
petitioners rest on the theory that they have acquired the property by acquisitive
prescription; and that Yap, without any right or authority, sold the same to LSBDA.
Conclusion
In the light of our earlier disquisition, the theory has no leg to stand on. Absent any
showing that the land has been classified as alienable, their possession thereof, no
matter how lengthy, cannot ripen into ownership. In other words, they have not
become owners of the disputed property. Moreover, LSBDA's title was derived from
a Miscellaneous Sales Patent, not from Yap. Finally, petitioners cannot, by a
collateral attack, challenge a certificate of title that has already become
indefeasible and incontrovertible.

If petitioners believe that they have been defrauded by Yap, they should seek
redress, not in these proceedings, but in a proper action in accordance with law.
WHEREFORE, the Petition is hereby DENIED and the assailed Decision AFFIRMED.
Costs against petitioners.
SO ORDERED.
Melo, Gonzaga-Reyes and Sandoval-Gutierrez, JJ., concur.
Vitug, J., concurs in the result.
||| (Seville v. National Development Co., G.R. No. 129401, [February 2, 2001], 403
PHIL 843-860)

SECOND DIVISION
[G.R. No. 73246. March 2, 1993.]
DIRECTOR OF LANDS AND DIRECTOR OF FOREST
DEVELOPMENT, petitioners, vs. INTERMEDIATE
APPELLATE
COURT
AND
J.
ANTONIO
ARANETA, respondents.
The Solicitor General for petitioners.
Jimenez, Leynes & Associates for private respondent.
SYLLABUS
1. LAND REGISTRATION; SUBMISSION OF TRACING CLOTH PLAN IS A MANDATORY
REQUIREMENT FOR REGISTRATION; SAME SHOULD BE RETRIEVED FROM THE LAND
REGISTRATION COMMISSION AND SUBMITTED IN EVIDENCE. In a very recent
decision of this Court, entitled The Director of Lands v. The Honorable Intermediate
Appellate Court and Lino Anit (G.R. No. No. 65663, October 16, 1992), We have
ruled that the submission of the tracing cloth plan is a mandatory requirement for
registration. Reiterating Our ruling in Director of Lands v. Reyes (68 SCRA 177), We
asserted that failure to submit in evidence the original tracing cloth plan is fatal it
being a statutory requirement of mandatory character. It is of no import that
petitioner failed to object to the presentation of the certified copy of the said plan.
What is required is the original tracing cloth plan of the land applied for and
objection to such requirement cannot be waived either expressly or impliedly. This
case is no different from the case of Director of Lands v. Reyes, supra wherein We
said that if the original tracing cloth plan was indeed with the Land Registration
Commission, there is no reason why the applicant cannot easily retrieve the same
and submit it in evidence, it being an essential requirement for registration.
2. ID.; PROHIBITION AGAINST PRIVATE CORPORATIONS OR ASSOCIATIONS ON
HOLDING OF ALIENABLE LANDS OF THE PUBLIC DOMAIN; EXCEPTION.
Our Constitution,whether the 1973 or 1987, prohibits private corporations or
associations from holding alienable lands of the public domain except by lease.
Apparently realizing such prohibition, respondent amended its application to
conform with the mandates of the law.
3. ID.; REPUBLICATION OF AN AMENDED APPLICATION, WHEN AND WHEN NOT
NECESSARY. We cannot go along with petitioners' position that the absence of
republication of an amended application for registration is a jurisdictional flaw. We
should distinguish. Amendments to the application may be due to change in parties
or substantial change in the boundaries or increase in the area of the land applied
for. In the former case, neither the Land Registration Act, as amended,

nor Presidential Decree No. 1529, otherwise known as the Property Registration
Decree, requires republication and registration may be allowed by the court at any
stage of the proceeding upon just and reasonable terms. On the other hand,
republication is required if the amendment is due to substantial change in the
boundaries or increase in the area of the land applied for.
4. ID.; ACT NO. 496 AND P.D. 1529 ARE EXISTING LAWS AND CAN STAND TOGETHER;
PURPOSE OF P.D. 1529. We do not see any relevant dispute in the lower court's
application of Presidential Decree No. 1529, instead of Act No. 496, in adjudicating
the land to the then applicant, assuming that the land involved is registrable. Both
laws are existing and can stand together. P.D. 1529 was enacted to codify the
various laws relative to registration of property, in order to facilitate effective
implementation of said laws.
5. ID.; CLASSIFICATION OF LANDS OF THE PUBLIC DOMAIN; IN WHOM AUTHORITY TO
CLASSIFY IS VESTED; EFFECT OF CLASSIFICATION; CAUSE OF CLASSIFICATION
STATEMENT OF THE REGALIAN DOCTRINE; EFFECT OF LACK OF CLASSIFICATION.
Lands of the public domain are classified under three main categories, namely:
Mineral, Forest and Disposable or Alienable Lands. Under the
CommonwealthConstitution,only agricultural lands were allowed to be alienated.
Their disposition was provided for under Commonwealth Act No. 141 (Secs. 6-7),
which states that it is only the President, upon the recommendation of the proper
department head, who has the authority to classify the lands of the public domain
into alienable or disposable, timber and mineral lands. Mineral and Timber or forest
lands are not subject to private ownership unless they are first reclassified as
agricultural lands and so released for alienation. In the absence of such
classification, the land remains as unclassified land until released therefrom and
rendered open to disposition. Courts have no authority to do so. This is in
consonance with the Regalian doctrine that all lands of the public domain belong to
the State, and that the State is the source of any asserted right to ownership in land
and charged with the conservation of such patrimony. Under the Regalian Doctrine,
all lands not otherwise appearing to be clearly within private ownership are
presumed to belong to the State. Hence, a positive act of the government is needed
to declassify a forest land into alienable or disposable land for agricultural or other
purposes.
6. ID.; POSSESSION, HOWEVER LONG, AND CONVERSION OF AN UNCLASSIFIED
PUBLIC LAND CANNOT RIPEN INTO PRIVATE OWNERSHIP. Respondent even
admitted that Tambac Island is still an unclassified public land as of 1927 and
remains to be unclassified. Since the subject property is still unclassified, whatever
possession the applicant may have had and however long, cannot ripen into private
ownership. The conversion of subject property does not automatically render the
property as alienable and disposable.
7. ID.; BURDEN OF PROOF AND EVIDENCE REQUIRED IN APPLICATION FOR
REGISTRATION OF LAND. The burden of proof in overcoming the presumption of
state ownership of the lands of the public domain is on the person applying for
registration that the land subject of the application is alienable or disposable. Unless
the applicant succeeds in showing by convincing evidence that the property
involved was acquired by him or his ancestors either by composition title from the
Spanish Government or by possessory information title, or any other means for the
proper acquisition of public lands, the property must be held to be part of the public

domain. The applicant must present evidence and persuasive proof to substantiate
his claim.
8. ID.; EVIDENTIARY WEIGHT OF TAX DECLARATIONS AND RECEIPTS. In any case
tax declarations and receipts are not conclusive evidence of ownership or of the
right to possess land when not supported by evidence. The fact that the disputed
property may have been declared for taxation purposes in the names of the
applicants or of their predecessors-in-interest way back in 1921 does not
necessarily prove ownership. They are merely indicia of a claim of ownership.

DECISION

NOCON, J p:
For review before Us is the decision of the Court of Appeals in the land registration
case entitled J. Antonio Araneta v. The Director of Lands and Director of Forest
Development, AC-G.R. CV No. 00636, 1 affirming the lower court's approval of the
application for registration of a parcel of land in favor of applicant therein, J. Antonio
Araneta.
Evidence show that the land involved is actually an island known as Tambac Island
in Lingayen Gulf. Situated in the Municipality of Bani, Pangasinan, the area consists
of 187,288 square meters, more or less. The initial application for registration was
filed for Pacific Farms, Inc. under the provisions of the Land Registration Act, Act No.
496, as amended. prcd
The Republic of the Philippines, thru the Director of Lands opposed the application
alleging that the applicant, Pacific Farms, Inc. does not possess a fee simple title to
the land nor did its predecessors possess the land for at least thirty (30) years
immediately preceding the filing of application. The opposition likewise specifically
alleged that the applicant is a private corporation disqualified under the (1973)
newPhilippine Constitution from acquiring alienable lands of the public domain
citing Section 11, Article 14. 2
The Director of Forest Development also entered its opposition alleging that the
land is within the unclassified public land and, hence, inalienable. Other private
parties also filed their oppositions, but were subsequently withdrawn.
In an amended application, Pacific Farms, Inc. filed a manifestation-motion to
change the applicant from Pacific Farms, Inc. to J. Antonio Araneta. Despite the
supposed amendment, there was no republication.
Evidence presented by the applicant include the testimony of Placido Orlando,
fishery guard of Pacific Farms, Inc., who said that he has known the disputed land
since he attained the age of reason for some forty (40) years now; that when he
first came to know the property it was then owned by and in the possession of
Paulino Castelo, Juan Ambrosio and Julio Castelo, and later on the whole island was
bought by Atty. Vicente Castelo who in turn sold it to J. Antonio Araneta.
Deposition by oral examination of Araneta was also presented, together with
documents of sale, tax declarations and receipts, and survey of property. Applicant,
however, failed to present the tracing cloth plan and instead submitted to the court
certified copies thereof.
While this case is pending here in Court, respondent filed an Omnibus Motion for
Substitution of private respondent. 3 Apparently, Antonio Araneta had assigned his

rights to and interest in Tambac Island to Amancio R. Garcia 4 who in turn assigned
his rights and interest in the same property to Johnny A. Khonghun whose
nationality was not alleged in the pleadings.
On October 4, 1979, the trial court rendered a decision adjudicating the subject
property to J. Antonio Araneta. On appeal to the then Intermediate Appellate Court,
the decision of the lower court was affirmed on December 12, 1985.
Petitioners raised the following errors:
I. The lower court erred in adjudicating the lands subject of
registration to applicant-appellee despite his failure to present the
original tracing cloth plan the submission of which is a statutory
requirement of mandatory character.

II. The lower court erred in not denying registration in favor of J.


Antonio Araneta since the amendment of the application was simply
an attempt to avoid the application of the constitutional provision
disqualifying a private corporation the Pacific Farms, Inc. in this
case from acquiring lands of public domain. cdrep
III. The lower court erred in not declaring the land known as the
"Tambac Island" not subject of registration it being an island formed
on the seas.
IV. The lower court erred in adjudicating the land to the applicant
under the provisions of Presidential Decree No. 1529, otherwise
known as the Property Registration Decree, despite absence of any
specific invocation of this law in the original and amended
application.
V. The lower court erred in not granting the government's motion for
reconsideration at least to enable it to present proof of the status of
the land as within the unclassified public forest, and hence beyond
the court's jurisdiction to adjudicate as private property.
VI. The lower court erred in not declaring that the applicant has failed
to overthrow the presumption that the land is a portion of the public
domain belonging to the Republic of the Philippines.
From the foregoing it appears that the more important issues are:
1) whether the presentation of the tracing cloth plan is necessary; and 2) whether
the land known as "Tambac Island" can be subject to registration.
By mere consideration of the first assignment of error, We can right away glean the
merit of the petition.
Respondent claims that the tracing cloth plan is with the files of the Land
Registration Commission, and the only evidence that can be presented to that fact
is the request for the issuance of a certified copy thereof and the certified copy
issued pursuant to the request. 5 Respondent further argues that failure of the
petitioners to object to the presentation of the certified copy of the tracing cloth

plan was the basis of the trial court's denial of petitioner's motion for
reconsideration.
In a very recent decision of this Court, entitled The Director of Lands v. The
Honorable Intermediate Appellate Court and Lino Anit, 6 We have ruled that the
submission of the tracing cloth plan is a mandatory requirement for registration.
Reiterating Our ruling in Director of Lands v. Reyes, 7 We asserted that failure to
submit in evidence the original tracing cloth plan is fatal it being a statutory
requirement of mandatory character.
It is of no import that petitioner failed to object to the presentation of the certified
copy of the said plan. What is required is the original tracing cloth plan of the land
applied for and objection to such requirement cannot be waived either expressly or
impliedly. 8 This case is no different from the case of Director of Lands v. Reyes,
supra wherein We said that if the original tracing cloth plan was indeed with the
Land Registration Commission, there is no reason why the applicant cannot easily
retrieve the same and submit it in evidence, it being an essential requirement for
registration.
As to the second assignment of error, We are inclined to agree with petitioners that
the amendment of the application from the name of Pacific Farms, Inc., as
applicant, to the name of J. Antonio Araneta, was a mere attempt to evade
disqualification. Our Constitution,whether the 1973 9 or 1987, 10 prohibits private
corporations or associations from holding alienable lands of the public domain
except by lease. Apparently realizing such prohibition, respondent amended its
application to conform with the mandates of the law. cdphil
However, We cannot go along with petitioners' position that the absence of
republication of an amended application for registration is a jurisdictional flaw. We
should distinguish. Amendments to the application may be due to change in parties
or substantial change in the boundaries or increase in the area of the land applied
for.
In the former case, neither the Land Registration Act, as amended, nor Presidential
Decree No. 1529, otherwise known as the Property Registration Decree, requires
republication and registration may be allowed by the court at any stage of the
proceeding upon just and reasonable terms. 11 On the other hand, republication is
required if the amendment is due to substantial change in the boundaries or
increase in the area of the land applied for.
As to the fourth assignment of error, We do not see any relevant dispute in the
lower court's application of Presidential Decree No. 1529, instead of Act No. 496, in
adjudicating the land to the then applicant, assuming that the land involved is
registrable. Both laws are existing and can stand together. P.D. 1529 was enacted to
codify the various laws relative to registration of property, in order to facilitate
effective implementation of said laws. 12
The third, fifth and sixth assignment of errors are likewise meritorious and shall be
discussed forthwith together.
Respondent asserts that contrary to the allegation of petitioners, the reports of the
District Land Officer of Dagupan City, Land Inspector Perfecto Daroy and
Supervising Land Examiner Teodoro P. Nieva show that the subject property is an

unclassified public land, not forest land. This claim is rather misleading. The report
of Supervising Land Examiner Nieva specifically states that the "land is within the
unclassified forest land" under the administrative jurisdiction of the then Bureau of
Forest Development. 13 This was based on the reports of Land Inspector Daroy and
District Land Officer Feliciano Liggayu.
Lands of the public domain are classified under three main categories, namely:
Mineral, Forest and Disposable or Alienable Lands. 14 Under the
Commonwealth Constitution,only agricultural lands were allowed to be alienated.
Their disposition was provided for under Commonwealth Act No. 141 (Secs. 6-7),
which states that it is only the President, upon the recommendation of the proper
department head, who has the authority to classify the lands of the public domain
into alienable or disposable, timber and mineral lands. Mineral and Timber or forest
lands are not subject to private ownership unless they are first reclassified as
agricultural lands and so released for alienation. 15 In the absence of such
classification, the land remains as unclassified land until released therefrom and
rendered open to disposition. Courts have no authority to do so. 16
This is in consonance with the Regalian doctrine that all lands of the public domain
belong to the State, and that the State is the source of any asserted right to
ownership in land and charged with the conservation of such patrimony. Under the
Regalian Doctrine, all lands not otherwise appearing to be clearly within private
ownership are presumed to belong to the State. Hence, a positive act of the
government is needed to declassify a forest land into alienable or disposable land
for agricultural or other purposes. 17
The burden of proof in overcoming the presumption of state ownership of the lands
of the public domain is on the person applying for registration that the land subject
of the application is alienable or disposable. 18
Unless the applicant succeeds in showing by convincing evidence that the property
involved was acquired by him or his ancestors either by composition title from the
Spanish Government or by possessory information title, or any other means for the
proper acquisition of public lands, the property must be held to be part of the public
domain. The applicant must present evidence and persuasive proof to substantiate
his claim. 19
In this particular case, respondent presented proof that as early as 1921, the
subject property has been declared for tax purposes with receipts attached, in the
names of respondent's predecessors-in-interest. Nevertheless, in that span of time
there had been no attempt to register the same either under Act 496 or under the
Spanish Mortgage Law. It is also rather intriguing that Vicente Castelo who acquired
almost 90% of the property from Alejo Ambrosio, et al. on June 18, 1958 and from
Julio Castelo on June 19, 1958 immediately sold the same to applicant J. Antonio
Araneta on 3 July 1958. LexLib
According to the report of Land Investigator Daroy, the land was declared for
taxation purposes in the name of Vicente Castelo only in 1958 and the purported
old tax declarations are not on file with the Provincial Assessor's Office.
In any case tax declarations and receipts are not conclusive evidence of ownership
or of the right to possess land when not supported by evidence. 20 The fact that the
disputed property may have been declared for taxation purposes in the names of

the applicants or of their predecessors-in-interest way back in 1921 does not


necessarily prove ownership. They are merely indicia of a claim of ownership. 21
Respondent's contention that the BFD, LC Map No. 681, certified on August 8, 1927
which was the basis of the report and recommendation of the Land Examiner, is too
antiquated; that it cannot be conclusively relied upon and was not even presented
in evidence, is not well taken. As We have said in the case of Director of Lands v.
CA: 22
"And the fact that BF Map LC No. 673 dated March 1, 1927 showing
subject property to be within unclassified region was not presented in
evidence will not operate against the State considering the
stipulation between the parties and under the well-settled rule that
the State cannot be estopped by the omission, mistake or error of its
officials or agents, if omission there was, in fact."
Respondent even admitted that Tambac Island is still an unclassified public land
as of 1927 and remains to be unclassified.
Since the subject property is still unclassified, whatever possession the applicant
may have had and however long, cannot ripen into private ownership. 23 The
conversion of subject property does not automatically render the property as
alienable and disposable.

In effect what the courts a quo have done is to release the subject property from the
unclassified category, which is beyond their competence and jurisdiction. We
reiterate that the classification of public lands is an exclusive prerogative of the
Executive Department of the Government and not of the Courts. In the absence of
such classification, the land remains unclassified until released therefrom and
rendered open to disposition. 24
In fairness to respondent, the petitioners should seriously consider the matter of the
reclassification of the land in question. The attempt of people to have disposable
lands they have been tilling for generations titled in their name should not only be
viewed with understanding attitude, but as a matter of policy encouraged. 25
WHEREFORE, the petition is hereby GRANTED and the decisions of the courts a
quo are REVERSED.
SO ORDERED.
Narvasa, C .J ., Padilla, Regalado and Campos, Jr., JJ ., concur.
||| (Director of Lands v. Intermediate Appellate Court, G.R. No. 73246, [March 2,
1993])

EN BANC
[G.R. No. 63786-87. April 7, 1993.]

UNITED PARACALE MINING COMPANY,


INC.,
AND
COCO
GROVE,
INC., petitioners, vs. HON.
JOSELITO
DELA ROSA, in his capacity as the
former Judge of the Court of First
Instance of Camarines Norte, Branch
2, et al., respondents.
[G.R. No. 70423. April 7, 1993.]
ZAMBALES
CHROMITE
MINING
COMPANY, INC., petitioner, vs. HON.
ALFREDO L. BENIPAYO, as Judge,
Regional Trial Court of Manila, and
PHILZEA MINING & DEVELOPMENT
CORPORATION, respondents.
[G.R. No. 73931. April 7, 1993.]
JOSEPH V. LOPEZ and MIGUEL C.
ANDRADE, petitioners, vs.
THE
INTERMEDIATE
APPELLATE
COURT
MARSMAN & COMPANY, INC. and
UNITED PARACALE MINING COMPANY,
INC., respondents.
Sycip, Salazar, Hernandez and Gatmaitan for United Paracale and Coco Grove, Inc.
Pedro A. Venida for respondents in G.R. 63786-87 and petitioners in G.R. 73931.
J.V. Natividad & Associates for Zambales Chromite.
SYLLABUS
1. STATUTORY CONSTRUCTION; INTERPRETATION OF A STATUTE; PROPER ONLY
WHEN THERE IS DOUBT OR AMBIGUITY IN ITS LANGUAGE; CASE AT BAR. The view
of the petitioner that by virtue of the registration of the mining claims under the
Philippine Bill of 1902 and Act No. 624, the mining claims became private property
and thereby brought outside the control and supervision of the Director of Mines is
without legal basis. The abovecited law does not distinguish between private
property and lands of the public domain. The provision of law involved is clear and
is not susceptible to interpretation. A condition sine qua non before the court may
construe or interpret a statute is that there be doubt or ambiguity in its language.
Section 7 of P.D. 1281 quoted above defining the original and exclusive jurisdiction

of the Director of Mines is clear. Time and again, it has been repeatedly declared by
this Court that where the law speaks in clear and categorical language, there is no
room for interpretation. There is only room for application. [Cebu Portland Cement
Company vs. Municipality of Naga, Cebu, 35 SCRA 708 (1968)] Where the law is
clear and unambiguous, it must be taken to mean exactly what it says and the court
has no choice but to see to it that its mandate is obeyed. [Chartered Bank
Employees Association vs. Ople, 138 SCRA 273 (1985)].
2. REMEDIAL LAW; CIVIL PROCEDURE; JUDICIAL RELIEF; NOT A VESTED RIGHT; A
MERE STATUTORY PRIVILEGE, NOT A PROPERTY RIGHT. There can be no vested
right in a judicial relief for this is a mere statutory privilege and not a property right.
The distinction between statutory privileges and vested rights must be borne in
mind for persons have no vested rights in statutory privileges. The state may
change or take away rights which were created by the law of the state, although it
may not take away property which was vested by virtue of such rights.(16A Am. Jur.
2d, pp. 652-653) Besides, the right to judicial relief is not a right which may
constitute vested right because to be vested, a right must have become a title,
legal or equitable, to the present or future enjoyment of property, or to the present
or future enforcement of a demand or legal exemption from a demand made by
another. (National Carloading Corporation v. Phoenix-E1 Paso Express, Inc., cited in
16A Am, Jur. 2d, p. 651) Definitely, the judicial relief herein referred to by the
petitioner does not fall under any of these.
3. CIVIL LAW; PRESIDENTIAL DECREE 1214; A VALID EXERCISE OF THE SOVEREIGN
POWER OF THE STATE AS OWNER OF LAND OF PUBLIC DOMAIN; SUSTAINED IN CASE
AT BAR. The heart of these twin petitions is the question of constitutionality
of P.D. 1214. Unless P.D. 1214 is successfully assailed, the petitioners will be but
mere applicants for the lease of the mining claims involved and would thus have no
causes of action against private respondents. This question has been resolved by
this Court in Santa Rosa Mining Company, Inc. vs. Leido, Jr. [156 SCRA 1 (1987),
which ruling was reiterated in Zambales Chromite Mining Company, Inc. vs. Leido,
Jr., 176 SCRA 602 (1989)] thus: "(W)e hold that Presidential Decree No. 1214 is not
unconstitutional.' It is a valid exercise of the sovereign power of the State, as
owner, over lands of the public domain, of which petitioner's mining claims still form
a part, and over the patrimony of the nation, of which mineral deposits are a
valuable asset. It may be underscored, in this connection, that the Decree does not
cover all mining claims located under the Phil. Bill of 1902, but only those claims
over which their locators had failed to obtain a patent. And even then, such locators
may still avail of the renewable twenty-five year (25) lease prescribed by Pres.
Decree No. 463, the Mineral Development Resources Decree of 1974. Mere location
does not mean absolute ownership over the affected land or the mining claim. It
merely segregates the located land or area from the public domain by barring other
would-be locators from locating the same and appropriating for themselves the
minerals found therein. To rule otherwise would imply that location is all that is
needed to acquire and maintain rights over a located mining claim. This, we cannot
approve or sanction because it is contrary to the intention of the lawmaker that the
locator should faithfully and consistently comply with the requirements for annual
work and improvements in the located mining claim. This, we cannot approve or
sanction because it is contrary to the intention of the lawmaker that the locator
should faithfully and consistently comply with the requirements for annual work and
improvements in the located mining claim.Presidential Decree No. 1214 is in accord

with Sec. 8, Art. XIV of the 1973 Constitution. The same Constitutional mandate is
found in Sec. 2, Art. XII of the 1987 Constitution.

DECISION

CAMPOS, JR., J p:
The cases herein were consolidated upon the representations of petitioners that
they involve the same issues or questions of law or at the very least, revolve around
the same set of facts. A perusal of the records, however, reveals the contrary. Only
two petitions are properly consolidated. Thus, it behooves Us to discuss the cases
separately.
In blatant violation of Section 2, Rule 45 of the Rules of Court which in part,
provides:
"Sec. 2. Contents of the petition The petition shall
contain a concise statement of the matters involved,
the assignment of errors made in the court below, and
the reasons relied on for the allowance of the
petition, . . . (Emphasis Ours.).
"Only questions of law may be raised in the petition
and must be distinctly set forth . . ."
this petition devotes nine (9) pages under the subtitle "Summary
statement of the matters involved" to a discussion of matters off
tangent from the real issues in the case. Definitely, the question
of whether or not the Court of Appeals erred in ruling that the
Regional Trial Court did not commit grave abuse of discretion in
issuing an order suspending hearing pending the resolution of
their motion to dismiss, does not involve the Philippine Bill of
1902, Executive Order No. 141, much less P.D. 1214. The counsel
for petitioners even discussed pending cases in this Court (G.R.
No. 63786 - 87 and 69203) which have completely nothing to do
with the instant petition except for the fact that the parties
therein are being represented by the same counsel as in this
petition. In several pleadings subsequent to their petition,
petitioners insisted that the proceedings in the court below must
be restrained until this Court resolves the pending cases
abovecited. For this reason this case was consolidated thereto.
A summary of the real matters involved in this petition is found in the respondent
Court's decision, to wit:

"This is a petition for certiorari and prohibition to


enjoin the Regional Trial Court, Branch XL, in
Camarines Norte from issuing a writ of preliminary
injunction in Civil Case No. 5148 and to disqualify the
respondent judge from acting in that case. The case
was brought by the respondents Marsman and Co.,
Inc. and United Paracale Mining, Inc., to enjoin the
petitioners, Joseph V. Lopez and Miguel Andrade, from
entering and conducting mining operations within the
"McDonald" and "San Antonio" Tunnels in Paracale,
Camarines Norte, in which the private respondents
have mining claims ("Tulisan," "Santa Marta,"
"California," and "Rocky Mountain Fraction"). On
December 11, 1984 the RTC issued a restraining order
against the petitioners. cdphil
On December 12 the petitioners filed their answer
alleging that, in accordance with PD 1214, the private
respondents had forfeited their right to the mining
claims. They likewise argued that in view of PD 605,
the RTC had no jurisdiction to entertain the case. On
the same day the petitioners moved for the
disqualification of the respondent judge of the RTC,
claiming (sic) that in issuing the restraining order of
December 11, 1984, he showed his "bias, prejudice
and personal hatred of and hostility to the
[petitioners'] counsel [Atty. Pedro A. Venida]."
On December 24, 1984 the petitioners filed a motion
for a preliminary hearing on their defense that the RTC
lacked jurisdiction under PD 605 to issue a temporary
restraining order or injunction in cases involving or
growing out of the action of administrative officials on
the applications for permits for the disposition,
exploitation, utilization, or exploration or development
of the natural resources. Accordingly the RTC, in its
order of September 5, 1985, suspended the hearing of
the case until the resolution of the petitioners' motion
to dismiss. It is at this point that the present petition
was filed." 1
The respondent Court denied this petition on grounds that: (1) the questions being
raised are not proper in a petition for certiorari under Rule 65 but rather defenses
which should be raised in the action itself; (2) the question of jurisdiction which has
yet to be resolved by the trial court pending resolution of the motion to dismiss is
prematurely raised; and (3) there was no basis for determining whether or not the
judge must be disqualified. 2

The review of this decision is what is on appeal before.

We refuse to be persuaded by the petitioners that the RTC must be enjoined from
exercising its jurisdiction in settling the case presented before it for the reason that
the constitutionality of the law involved in the said case is being questioned before
this Court. This case should have been disposed of independently of the other
petitions herein.
The respondent Court of Appeals committed no reversible error. Neither did it
commit grave abuse of discretion as what petitioners want this Court to believe. The
petitioners fail to point out any assigned error which the respondent Court had
supposedly committed but simply narrate the action taken by it. Much less have
they stated the reasons relied upon for the allowance of the instant petition. For
being insufficient in substance and in form, the instant petition lacks merit and must
be dismissed.
G.R. No. 70423
This is a petition involving the question of jurisdiction of regular courts in cases
which had been placed under the original and exclusive jurisdiction of the Bureau of
Mines underP.D. 1281.
This petition seeks to reverse the order of then Judge, now Associate Justice of the
Court of Appeals, Hon. Alfredo L. Benipayo, dismissing the complaint filed by
petitioner herein on the ground of lack of jurisdiction, citing Section 7 of P.D.
1281 and the doctrine enunciated in Twin Peaks Mining Association, et al. vs.
Navarro, 3 that an action for the enforcement of mining contracts, in this case
cancellation of a mining contract, is outside the competence of regular courts in
view of the law cited. 4
The complaint filed with the then CFI of Manila, Branch XVI, was one for the
rescission of its mining contract with herein private respondent on grounds of
violations of the terms and conditions thereof, with prayer for the issuance of a
preliminary injunction and/or temporary restraining order. The trial court, however,
upon motion of the defendant therein, dismissed the case.
Petitioner wants Us to construe Section 7 of P.D. 1281 as applicable only to mineral
lands forming part of the public domain and not to mining claims located and
registered under Philippine Bill of 1902 and Act No. 624 as is its case.
Section 7 of P.D. 1281 reads as follows:
Sec. 7. In addition to its regulatory and adjudicative
functions over companies, partnerships or persons
engaged in mining exploration, development and
exploitation, the Bureau of Mines shall have original
and exclusive jurisdiction to hear and decide cases
involving:
(a) a mining property subject of different
agreements entered into by the claim holder
thereof with several mining operators;
(b) complaints from claimowners that the
mining property subject of an operating
agreement has not been placed into actual

operations within the period stipulated therein;


and
(c) cancellation and/or reinforcement of
mining contracts due to the refusal of the
claimowner/operator to aside by the terms and
conditions thereof.
All actions and decisions of the Director of Mines on
the above cases are subject to review, motu proprio or
upon appeal by any person aggrieved thereby, by the
Secretary of Natural Resources whose decision shall
be final and executory after the lapse of thirty (30)
days from receipt by the aggrieved party of said
decision, unless appealed to the President in
accordance with the applicable provisions
of Presidential Decree No. 309 and Letter of
Instructions Nos. 119 and 135. LLpr
The view of the petitioner that by virtue of the registration of the mining claims
under the Philippine Bill of 1902 and Act No. 624, the mining claims became private
property and thereby brought outside the control and supervision of the Director of
Mines is without legal basis. The abovecited law does not distinguish between
private property and lands of the public domain. The provision of law involved is
clear and is not susceptible to interpretation. A condition sine qua non before the
court may construe or interpret a statute is that there be doubt or ambiguity in its
language. 5 Section 7 of P.D. 1281 quoted above defining the original and exclusive
jurisdiction of the Director of Mines is clear. Time and again, it has been repeatedly
declared by this Court that where the law speaks in clear and categorical language,
there is no room for interpretation. There is only room for application. 6 Where the
law is clear and unambiguous, it must be taken to mean exactly what it says and
the court has no choice but to see to it that its mandate is obeyed. 7
This Court in Benguet Corporation vs. Leviste, 8 made these pronouncements:
"We grant the petition. Presidential Decree No.
1281 which took effect on January 16, 1978 vests the
Bureau of Mines with jurisdictional supervision and
control over all holders of mining claims or applicants
for and/or grantees of mining licenses, permits, leases
and/or operators thereof, including mining service
contracts and service contractors insofar as their
mining activities are concerned. To effectively
discharge its task as the Government's arm in the
administration and disposition of mineral resources,
Section 7 of P.D. 1281 confers upon the Bureau quasijudicial powers as follows:
xxx xxx xxx
Analyzing the objective of P.D. 1281, particularly said
Section 7 thereof, the Court in Twin Peaks Mining
Association, the case relied upon by petitioner, noted

that the trend is to make the adjudication of mining


cases a purely administrative matter. This observation
was reiterated in the more recent case of Atlas
Consolidated Mining & Development Corporation vs.
Court of Appeals."
The petitioner further argues that to hold that P.D. 1281 retroactively applies to its
mining claims which according to it is private property would constitute impairment
of vested rights since by shifting the forum of the petitioner's case from the courts
to the Bureau of Mines, as urged by private respondent, the substantive rights to
full protection of its property rights shall be greatly impaired and prejudiced. The
judicial relief available for the redress of private property rights violated, now being
enjoyed by petitioner shall be lost altogether.
This argument does not merit Our approval. There can be no vested right in a
judicial relief for this is a mere statutory privilege and not a property right. The
distinction between statutory privileges and vested rights must be borne in mind for
persons have no vested rights in statutory privileges. The state may change or take
away rights which were created by the law of the state, although it may not take
away property which was vested by virtue of such rights. 9 Besides, the right to
judicial relief is not a right which may constitute vested right because to be vested,
a right must have become a title, legal or equitable, to the present or future
enjoyment of property, or to the present or future enforcement of a demand or legal
exemption from a demand made by another. 10 Definitely, the judicial relief herein
referred to by the petitioner does not fall under any of these.
The case at bar falls within the original and exclusive jurisdiction of the Bureau of
Mines, hence, the trial court did not err in dismissing the petitioner's complaint on
the ground of lack of jurisdiction.
G.R. Nos. 63786-87
In these petitions filed by petitioners United Paracale Mining Company, Inc. and
Coco Grove, Inc., petitioners seek to set aside the Order of dismissal of the case
they filed with the trial court for the ejectment of their respective defendants from
the mining claims which were allegedly privately owned by them having been
located and perfected under the provisions of the Philippine Bill of 1902 and Act No.
624.
The heart of these twin petitions is the question of constitutionality of P.D. 1214.
Unless P.D. 1214 is successfully assailed, the petitioners will be but mere applicants
for the lease of the mining claims involved and would thus have no causes of action
against private respondents.
This question has been resolved by this Court in Santa Rosa Mining Company, Inc.
vs. Leido, Jr. 11 thus:
"(W)e hold that Presidential Decree No. 1214 is not
unconstitutional. ** It is a valid exercise of the
sovereign power of the State, as owner, over lands of
the public domain, of which petitioner's mining claims
still form a part, and over the patrimony of the nation,
of which mineral deposits are a valuable asset. It may
be underscored, in this connection, that the Decree

does not cover all mining claims located under the


Phil. Bill of 1902, but only those claims over which
their locators had failed to obtain a patent. And even
then, such locators may still avail of the renewable
twenty-five year (25) lease prescribed by Pres. Decree
No. 463, the Mineral Development Resources Decree
of 1974.
Mere location does not mean absolute ownership over
the affected land or the mining claim. It merely
segregates the located land or area from the public
domain by barring other would-be locators from
locating the same and appropriating for themselves
the minerals found therein. To rule otherwise would
imply that location is all that is needed toacquire and
maintain rights over a located mining claim. This, we
cannot approve or sanction because it is contrary to
the intention of the lawmaker that the locator should
faithfully and consistently comply with the
requirements for annual work and improvements in
the located mining claim.
Presidential Decree No. 1214 is in accord with Sec. 8,
Art. XIV of the 1973 Constitution which states:
'All lands of the public domain, waters,
minerals, coal, petroleum, and other mineral
oils, all forces of potential energy, fisheries,
wildlife, and other natural resources of the
Philippines belong to the State. With the
exception of agricultural, industrial or
commercial, residential and resettlement lands
of the public domain, natural resources shall not
be alienated, and no license, concession, or
lease for the exploration, development, and
exploitation, or utilization of any of the natural
resources shall be granted for a period
exceeding twenty-five years, renewable for not
more than twenty-five years, except as to water
rights for irrigation, water supply, fisheries, or
industrial uses other than development of water
power, in which cases, beneficial use may be
the measure and the limit of the grant.'

The same Constitutional mandate is found in Sec. 2,


Art. XII of the 1987 Constitution, which declares: LLjur
'All lands of the public domain, waters,
minerals, coal, petroleum, and other mineral
oils, all forces of potential energy, fisheries,
forests or timber, wildlife, flora and fauna, and

other natural resources are owned by the State.


With the exception of agricultural lands, all
other natural resources shall not be alienated.
The exploration, development, and utilization of
natural resources shall be under the full control
and supervision of the State . . .'"
Notwithstanding Our ruling , in favor of the constitutionality of P.D. 1214, petitioners
contend that having filed mining lease applications on the mining claims they have
previously located and registered under then existing laws, pursuant to the
requirements of this Presidential Decree, and despite the waiver of their rights to
the issuance of mining patents therefor (emphasis theirs), they cannot be placed in
equal footing with those who forfeit all rights by virtue of non-filing of an application
within the prescribed period such that they (petitioners) have no causes of action
against private respondents.
We are not persuaded by this contention.
Although We may agree that those who filed their mining lease applications have
better rights than those who forfeited all their right by not filing at all, this, however,
does not amount to any vested right which could be the basis for their cause of
action against herein private respondents. What is precisely waived is their right to
the issuance of a mining patent upon application. This in effect grants the
government the power, in the exercise of its sound discretion, to award the patent
to the applicant most qualified to explore, develop and exploit the mineral resources
of the country in line with the objectives of P.D. 463, and not necessarily to the
original locator of the mining claim. To sustain their contention that they can
question the award of mining patents to applicants other than them would put to
naught the objectives of P.D. 1214 as enunciated in its WHEREASclauses.
We agree with the trial court that with the waiver of their right to the issuance of a
mining patent upon their application for a mining lease, their status is reduced to a
mere applicant, their only advantage over the others is the fact that they have
already conducted explorations at the site and this exploration may be ongoing. But
still, this credential, so to speak, is not intended to tie the hands of the government
so as to prevent it from awarding the mining patent to some other applicants, which
in its belief may be more qualified than them.
WHEREFORE, the petition in G.R. No. 73931 is hereby DISMISSED for lack of merit;
the Order of dismissal assailed in G.R. No. 70423 is AFFIRMED and this petition is
hereby likewise DISMISSED; the Order of dismissal assailed in G.R. Nos. 63786-87 is
AFFIRMED and these petitions are hereby DISMISSED. No pronouncements as to
costs.
SO ORDERED.
Narvasa, C .J ., Cruz, Padilla, Bidin, Grio-Aquino, Regalado, Davide, Jr., Romero,
Nocon, Bellosillo, Melo and Quiason, JJ ., concur.
Feliciano, J ., took no part.

||| (United Paracale Mining Co., Inc. v. Dela Rosa, G.R. No. 63786-87, 70423, 73931,
[April 7, 1993])

FIRST DIVISION
[G.R. No. 128017. January 20, 1999.]
RAMON ITURALDE, petitioner, vs. ALFREDO
FALCASANTOS, respondent.
Leo Jay T. Principe for petitioner.
Bienvenido G. Martin for private respondent.
SYNOPSIS
Petitioner acquired by purchase a 6-hectare land located at Baas, Lantawan,
Basilan Province on October 17, 1986. However, said lot was applied with the
Bureau of Lands by respondent for a free patent. The same was dismissed by the
Regional Director of Lands for failure to exercise the right to repurchase and allowed
petitioner to file a public land application for the subject land. Thereafter, petitioner
filed a complaint for recovery of ownership and possession against respondent. The
trial court rendered judgment in favor of petitioner who was declared the owner and
possessor of the subject land. On appeal, the Court of Appeals reversed the
appealed decision in finding that the land is within the forest reserve area, hence,
not capable of private appropriation and occupation. Hence, this recourse,
petitioner by claiming that allowance of the Director of Lands to file a public land
application for said property is equivalent to a declaration that said land was no
longer part of the public domain.
Lands within the forest reserve are not capable of private appropriation and
occupation; that a positive act of the government is needed to declassify a forest
land into alienable or disposable land for agricultural or other purposes; and that
possession of forest lands, however long, cannot ripen into private
ownership. ICHcTD
SYLLABUS
1. CIVIL LAW; PUBLIC LAND ACT; LANDS WITHIN FOREST RESERVE, NOT CAPABLE OF
PRIVATE APPROPRIATION AND OCCUPATION. The Court of Appeals correctly held
that "the evidence is unrebutted that the subject land is within the Forest Reserve
Area as per L.C. Map No. 1557 certified on August 13, 1951." and, hence, not
capable of private appropriation and occupation. In Republic vs. Register of Deeds
of Quezon, we held that "Forest lands, like mineral or timber lands which are public
lands, are not subject to private ownership unless they under the Constitution,
become private properties. In the absence of such classification, the land remains

unclassified public land until released therefrom and rendered open to


disposition." HDTCSI
2. ID.; ID.; DECLASSIFICATION OF PUBLIC LANDS; POSITIVE ACT FROM
GOVERNMENT, INDISPENSABLE. In Sunbeam Convenience Foods Inc. vs. Court of
Appeals, we said: "Thus, before any land may be declassified from the forest group
and converted into alienable or disposable land for agricultural or other purposes,
there must be a positive act from the government. Even rules on the confirmation of
imperfect titles do not apply unless and until the land classified as forest land is
released in an official proclamation to that effect so that it may form part of the
disposable agricultural lands of the public domain." Hence, a positive act of the
government is needed to declassify a forest land into alienable or disposable land
for agricultural or other purposes."
3. ID.; POSSESSION; POSSESSION OF FOREST LANDS HOWEVER LONG CANNOT
RIPEN INTO PRIVATE OWNERSHIP. The rule is "Possession of forest lands, however
long, cannot ripen into private ownership."

DECISION

PARDO, J p:
The case is an appeal via certiorari from a decision of the Court of Appeals reversing
that of the Regional Trial Court, Branch 2, Basilan province, and dismissing
petitioner's complaint for recovery of possession and ownership of a parcel of land
with the improvements existing thereon, situated at Barangay Upper Baas,
municipality of Lantawan, province of Basilan, with an area of 7.1248
hectares. cdlex
The facts may be related as follows:
On October 17, 1986, petitioner acquired by purchase from the heirs of Pedro Manaay a parcel of land located at Baas, Lantawan, Basilan Province, with an area of
6.0000 hectares, more or less, more particularly described as follows:
"A parcel of land, situated at Baas, Lantawan Basilan. Bounded on
the North by property of Alejandro Marso; on the East by property of
Ramon Bacor; on the South by property of Atty. Ricardo G. Mon and
on the West by property of Librada Guerrero. Containing an area of
6.0000 hectares, more or less."
However, on November 3, 1986, respondent applied with the Bureau of Lands in
Isabela, Basilan province, for the award to him of the same parcel of land under free
patent. On November 17, 1986, petitioner filed a protest to such application.
On February 7, 1989, the Regional Director of Lands rendered a decision giving
respondent a period of one hundred twenty (120) days to exercise the right to
repurchase the land by reimbursing petitioner of all expenses he incurred in the
purchase of the property in question, and held in abeyance respondent's application
for free patent.

On October 11, 1989, the Regional Director issued an order declaring that
respondent had waived his right of repurchase, and rejected his application for free
patent for lack of interest, and allowed petitioner to file a public land application for
the subject land.
On May 8, 1990, the Regional Director ordered respondent to vacate the land in
question, but respondent refused.
On July 24, 1990, petitioner filed with the Regional Trial Court, Basilan province, a
complaint for recovery of ownership and possession with preliminary injunction of
the subject parcel of land.
In answer to the complaint, respondent alleged that the land occupied by him
belonged to the Republic of the Philippines, and that he had introduced
improvements thereon such as coconut and other fruit trees. cdtai
After trial on the merits, on March 20, 1993, the trial court rendered decision
declaring petitioner the owner and possessor of the subject parcel of land with all
the improvements existing thereon, situated at Barangay Upper Baas, municipality
of Lantawan, province of Basilan, with an area of 3.1248 hectares, and ordering
respondent to vacate the land in question, to pay petitioner the amount of ten
thousand pesos (P10,000.00) as attorneys fee, the amount of five thousand pesos
(P5,000.00) as litigation expenses, and three hundred pesos (P300.00) as judicial
cost.
In due time, petitioner appealed the trial court's decision to the Court of Appeals.
On December 20, 1996, the Court of Appeals rendered decision reversing the
appealed decision, and entering a new judgment dismissing petitioner's complaint
without prejudice to any action that petitioner may take if the subject land was
declassified from forest land to alienable and disposable land of the public domain.
Hence, the present recourse.
Petitioner submits that the Court of Appeals erred in setting aside the trial court's
decision in his favor and dismissing the complaint because when the Director of
Lands allowed petitioner to file a public land application for said property, it was
equivalent to a declaration that said land was no longer part of the public domain.
We deny the petition. The Court of Appeals correctly held that "the evidence is
unrebutted that the subject land is within the Forest Reserve Area as per L.C. Map
No. 1557 certified on August 13, 1951' " 1 and, hence, not capable of private
appropriation and occupation. 2
In Republic vs. Register of Deeds of Quezon, we held that "Forest lands, like mineral
or timber lands which are public lands, are not subject to private ownership unless
they under the Constitution, become private properties. In the absence of such
classification, the land remains unclassified public land until released therefrom and
rendered open to disposition." 3
In Sunbeam Convenience Foods Inc. vs. Court of Appeals, we said: "Thus, before any
land may be declassified from the forest group and converted into alienable or
disposable land for agricultural or other purposes, there must be a positive act from
the government. Even rules on the confirmation of imperfect titles do not apply
unless and until the land classified as forest land is released in an official

proclamation to that effect so that it may form part of the disposable agricultural
lands of the public domain." 4
"Hence, a positive act of the government is needed to declassify a forest land into
alienable or disposable land for agricultural or other purposes." 5
And the rule is "Possession of forest lands, however long, cannot ripen into private
ownership." 6
What is more, there is yet no award or grant to petitioner of the land in question by
free patent or other ways of acquisition of public land. Consequently, he can not
lawfully claim to be the owner of the land in question.
WHEREFORE, the Court hereby AFFIRMS the appealed decision of the Court of
Appeals in CA-G.R. CV No. 42306, dismissing the complaint of petitioner before the
Regional Trial Court, Basilan province, in Civil Case No. 441-63. cdrep
No costs.
SO ORDERED.
Davide, Jr., C.J., Melo, Kapunan and Martinez, JJ., concur.
||| (Ituralde v. Falcasantos, G.R. No. 128017, [January 20, 1999], 361 PHIL 245-250)

FIRST DIVISION
[G.R. No. L-30389. December 27, 1972.]
PEDRO LEE HONG HOK, SIMEON LEE HONG HOK,
ROSITA LEE HONG HOK and LEONCIO LEE HONG
HOK, petitioners, vs. ANIANO
SECRETARY
RESOURCES,

OF

DAVID,

AGRICULTURE

THE

DIRECTOR

AND
OF

COURT OF APPEALS, respondents.

Augusto A. Pardalis for petitioners.


Luis General, Jr. for respondent Aniano David.

THE

HON.

NATURAL

LANDS

and

Office of the Solicitor General for other respondents.

DECISION

FERNANDO, J p:
Petitioners 1 in this appeal by certiorari would have us reverse a decision of
respondent Court of Appeals affirming a lower court judgment dismissing their
complaint to have the Torrens Title 2 of respondent Aniano David declared null and
void. What makes the task for petitioners quite difficult is that their factual support
for their pretension to ownership of such disputed lot through accretion was
rejected by respondent Court of Appeals. Without such underpinning, they must
perforce rely on a legal theory, which, to put it mildly, is distinguished by
unorthodoxy and is therefore far from persuasive. A grant by the government
through the appropriate public officials 3 exercising the competence duly vested in
them by law is not to be set at naught on the premise, unexpressed but implied,
that land not otherwise passing into private ownership may not be disposed of by
the state. Such an assumption is at war with settled principles of constitutional law.
It cannot receive our assent. We affirm.
The decision of respondent Court of Appeals following that of the lower court makes
clear that there is no legal justification for nullifying the right of respondent Aniano
David to the disputed lot arising from the grant made in his favor by respondent
officials. As noted in the decision under review, he "acquired lawful title thereto
pursuant to his miscellaneous sales application in accordance with which an order of

award and for issuance of a sales patent was made by the Director of Lands on June
18, 1958, covering Lot 2892 containing an area of 226 square meters, which is a
portion of Lot 2863 of the Naga Cadastre. On the basis of the order of award of the
Director of Lands the Undersecretary of Agriculture and Natural Resources issued on
August 26, 1959, Miscellaneous Sales Patent No. V-1209 pursuant to which OCT No.
510 was issued by the Register of Deeds of Naga City to defendant-appellee Aniano
David on October 21, 1959. According to the Stipulation of Facts, since the filing of
the sales application of Aniano David and during all the proceedings in connection
with said application, up to the actual issuance of the sales patent in his favor, the
plaintiffs-appellants did not put up any opposition or adverse claim thereto. This is
fatal to them because after the registration and issuance of the certificate and
duplicate certificate of title based on a public land patent, the land covered thereby
automatically comes under the operation of Republic Act 496 subject to all the
safeguards provided therein . . . Under Section 38 of Act 496 any question
concerning the validity of the certificate of title based on fraud should be raised
within one year from the date of the issuance of the patent. Thereafter the
certificate of title based thereon becomes indefeasible . . . In this case the land in
question is not a private property as the Director of Lands and the Secretary of
Agriculture and Natural Resources have always sustained the public character
thereof for having been formed by reclamation. . . . The only remedy therefore,
available to the appellants is an action for reconveyance on the ground of fraud. In
this case we do not see any fraud committed by defendant-appellant Aniano David
in applying for the purchase of the land involved through his Miscellaneous Sales
Application No. MSA-V-26747, entered in the records of the Bureau of Lands

[Miscellaneous Sales] Entry No. V-9033, because everything was done in the open.
The notices regarding the auction sale of the land were published, the actual sale
and award thereof to Aniano David were not clandestine but open and public official
acts of an officer of the Government. The application was merely a renewal of his
deceased wife's application, and the said deceased occupied the land since
1938." 4
On such finding of facts, the attempt of petitioners to elicit a different conclusion is
likely to be attended with frustration. The first error assigned predicated an
accretion having taken place, notwithstanding its rejection by respondent Court of
Appeals, would seek to disregard what was accepted by respondent Court as to how
the disputed lot came into being, namely by reclamation. It does not therefore call
for any further consideration. Neither of the other two errors imputed to respondent
Court, as to its holding that authoritative doctrines preclude a party other than the
government to dispute the validity of a grant and the recognition of the indefeasible
character of a public land patent after one year, is possessed of merit.
Consequently, as set forth at the outset, there is no justification for reversal.
1. More specifically, the shaft of criticism was let loose by petitioner aimed at this
legal proposition set forth in the exhaustive opinion of then Justice Salvador
Esguerra of the Court of Appeals, now a member of this Court: "There is,
furthermore, a fatal defect of parties to this action. Only the Government,
represented by the Director of Lands, or the Secretary of Agriculture and Natural
Resources, can bring an action to cancel a void certificate of title issued pursuant to
a void patent (Lucas vs. Durian, 102 Phil. 1157; Director of Lands vs. Heirs of Ciriaco

Carlo, G.R. No. L-12485, July 31, 1959). This was not done by said officers but by
private parties like the plaintiffs, who cannot claim that the patent and title issued
for the land involved are void since they are not the registered owners thereof nor
had they been declared as owners in the cadastral proceedings of Naga Cadastre
after claiming it as their private property. The cases cited by appellants are not in
point as they refer to private registered lands or public lands over which vested
rights have been acquired but notwithstanding such fact the Land Department
subsequently granted patents to public land applicants." 5 Petitioner ought to have
known better. The above excerpt is invulnerable to attack. It is a restatement of a
principle that dates back to Maninang v. Consolacion, 6 a 1908 decision. As was
there categorically stated: "The fact that the grant was made by the government is
undisputed Whether the grant was in conformity with the law or not is a question
which the government may raise, but until it is raised by the government and set
aside, the defendant can not question it. The legality of the grant is a question
between the grantee and the government." 7 The above citation was
repeated ipsissimis verbis in Salazar v. Court of Appeals. 8 Bereft as petitioners
were of the right of ownership in accordance with the findings of the Court of
Appeals, they cannot, in the language of Reyes v. Rodriguez, 9 "question the [title]
legally issued." 10 The second assignment of error is thus disposed of.
2. As there are overtones indicative of skepticism, if not of outright rejection, of the
well-known distinction in public law between the government authority possessed
by the state which is appropriately embraced in the concept of sovereignty, and its
capacity to own or acquire property, it is not inappropriate to pursue the matter
further. The former comes under the heading of imperium and the latter

ofdominium. The use of this term is appropriate with reference to lands held by the
state in its proprietary character. In such capacity, it may provide for the
exploitation and use of lands and other natural resources, including their
disposition, except as limited by the Constitution. Dean Pound did speak of the
confusion that existed during the medieval era between such two concepts, but did
note the existence ofres publicae as a corollary to dominium. 11 As far as the
Philippines was concerned, there was a recognition by Justice Holmes in Cario v.
Insular Government, 12 a case of Philippine origin, that "Spain in its earlier decrees
embodied the universal feudal theory that all lands were held from the
Crown . . ." 13 That was a manifestation of the concept of jura regalia, 14 which
was adopted by the present Constitution, ownership however being vested in the
state as such rather than the head thereof. What was stated by Holmes served to
confirm a much more extensive discussion of the matter in the leading case of
Valenton v. Murciano, 15 decided in 1904. One of the royal decrees cited was
incorporated in the Recopilacion de Leyes de las Indias 16 in these words: "We
having acquired in sovereignty over the Indies, and all lands, territories, and
possessions not heretofore ceded away by our royal predecessors, or by us, or in
our name, still pertaining to the royal crown and patrimony, it is our will that all
lands which are held without proper and true deeds of grant be restored to us
according as they belong to us, in order that after reserving before all what to us or
to our viceroys, audiencias, and governors may seem necessary for public squares,
ways, pastures, and commons in those places which are peopled, taking into
consideration not only their present condition, but also their future and their
probable increase, and after distributing to the natives what may be necessary for

tillage and pasturage, confirming them in what they now have and giving them
more if necessary, all the rest of said lands may remain free and unencumbered for
us to dispose of as we may wish." 17
It could therefore be affirmed in Montano v. Insular Government 18 that "as to the
unappropriated public lands constituting the public domain the sole power of
legislation is vested in Congress, . . ." 19 They continue to possess that character
until severed therefrom by state grant. 20 Where, as in this case, it was found by
the Court of Appeals that the disputed lot was the result of reclamation, its being
correctly categorized as public land is undeniable. 21 What was held in Heirs of
Datu Pendatun v. Director of Lands 22 finds application. Thus: "There being no
evidence whatever that the property in question was ever acquired by the
applicants or their ancestors either by composition title from the Spanish
Government or by possessory information title or by any other means for the
acquisition of public lands, the property must be held to be public domain." 23 For it
is well-settled "that no public land can be acquired by private persons without any
grant, express or implied, from the government." 24 It is indispensable then that
there be a showing of a title from the state or any other mode of acquisition
recognized by law. 25 The most recent restatement of the doctrine, found in an
opinion of Justice J.B.L. Reyes, follows: 26 "The applicant, having failed to establish
his right or title over the northern portion of Lot No. 463 involved in the present
controversy, and there being no showing that the same has been acquired by any
private person from the Government, either by purchase or by grant, the property is
and remains part of the public domain." 27 To repeat, the second assignment of
error is devoid of merit.

3. The last error assigned would take issue with this portion of the opinion of Justice
Esguerra: "According to the Stipulation of Facts, since the filing of the sales
application of Aniano David and during all the proceedings in connection with said
application, up to the actual issuance of the sales patent in his favor, the plaintiffsappellants did not put up any opposition or adverse claim thereto. This is fatal to
them because after the registration and issuance of the certificate and duplicate
certificate of title based on a public land patent, the land covered thereby
automatically comes under the operation of Republic Act 496 subject to all the
safeguards provided therein . . . Under Section 38 of Act 496 any question
concerning the validity of the certificate of title based on fraud should be raised
within one year from the date of the issuance of the patent. Thereafter the
certificate of title based thereon becomes indefeasible . . ." 28 Petitioners cannot
reconcile themselves to the view that respondent David's title is impressed with the
quality of indefeasibility. In thus manifesting such an attitude, they failed to accord
deference to controlling precedents. As far back as 1919, in Aquino v. Director of
Lands, 29 Justice Malcolm, speaking for the Court, stated: "The proceedings under
the Land Registration Law and under the provisions of Chapter VI of the Public Land
Law are the same in that both are against the whole world, both take the nature of
judicial proceedings, and for both the decree of registration issued is conclusive and
final." 30 Such a view has been followed since then. 31 The latest case in point is
Cabacug v. Lao. 32 There is this revealing excerpt appearing in that decision: "It is
said, and with reason, that a holder of a land acquired under a free patent is more
favorably situated than that of an owner of registered property. Not only does a free

patent have a force and effect of a Torrens Title, but in addition the person to whom
it is granted has likewise in his favor the right to repurchase within a period of five
years." 33 It is quite apparent, therefore, that petitioners' stand is legally
indefensible.
WHEREFORE, the decision of respondent Court of Appeals of January 31, 1969 and
its resolution of March 14, 1969 are affirmed. With costs against petitionersappellants.
Concepcion, C.J., Makalintal, Zaldivar, Castro, Teehankee, Barredo, Makasiar,
Antonioand Esguerra, JJ., concur.
||| (Lee Hong Hok v. David, G.R. No. L-30389, [December 27, 1972], 150-C PHIL 542550)

Potrebbero piacerti anche